Med Surg 3

Réussis tes devoirs et examens dès maintenant avec Quizwiz!

A nurse in the emergency department (ED) is triaging a 5-year-old who has been brought to the ED by her parents for an outbreak of urticaria. What would be the most appropriate question to ask this patient and her family? A) Has she eaten any new foods today? B) Has she bathed in the past 24 hours? C) Did she go to a friends house today? D) Was she digging in the dirt today?

A

A nurse is caring for a patient who has just had an arthroscopy as an outpatient and is getting ready to go home. The nurse should teach the patient to monitor closely for what postprocedure complication? A) Fever B) Crepitus C) Fasciculations D) Synovial fluid leakage

A

A clinic nurse is caring for a patient with a history of osteoporosis. Which of the following diagnostic tests best allows the care team to assess the patients risk of fracture? A) Arthrography B) Bone scan C) Bone densitometry D) Arthroscopy

C

A new patient has come to the dermatology clinic to be assessed for a reddened rash on his abdomen. What diagnostic test would most likely be ordered to identify the causative allergen? A) Skin scrapings B) Skin biopsy C) Patch testing D) Tzanck smear

C

A new patient presents at the clinic and the nurse performs a comprehensive health assessment. The nurse notes that the patients fingernail surfaces are pitted. The nurse should suspect the presence of what health problem? A) Eczema B) Systemic lupus erythematosus (SLE) C) Psoriasis D) Chronic obstructive pulmonary disease (COPD)

C

A nurse in a dermatology clinic is reading the electronic health record of a new patient. The nurse notes that the patient has a history of a primary skin lesion. What is an example of a primary skin lesion? A) Crust B) Keloid C) Pustule D) Ulcer

C

A nurse is caring for a patient who has been scheduled for a bone scan. What should the nurse teach the patient about this diagnostic test? A) The test is brief and requires that you drink a calcium solution 2 hours before the test. B) You will not be allowed fluid for 2 hours before and 3 hours after the test. C) Youll be encouraged to drink water after the administration of the radioisotope injection. D) This is a common test that can be safely performed on anyone.

C

A nurse is caring for a patient with a diagnosis of cancer that has metastasized. What laboratory value would the nurse expect to be elevated in this patient? A) Bilirubin B) Potassium C) Alkaline phosphatase D) Creatinine

C

A nurse is reviewing gerontologic considerations relating to the care of patients with dermatologic problems. What vulnerability results from the age-related loss of subcutaneous tissue? A) Decreased resistance to ultraviolet radiation B) Increased vulnerability to infection C) Diminished protection of tissues and organs D) Increased risk of skin malignancies

C

A nurse is taking a health history on a new patient who has been experiencing unexplained paresthesia. What question should guide the nurses assessment of the patients altered sensations? A) How does the strength in the affected extremity compare to the strength in the unaffected extremity? B) Does the color in the affected extremity match the color in the unaffected extremity? C) How does the feeling in the affected extremity compare with the feeling in the unaffected extremity? D) Does the patient have a family history of paresthesia or other forms of altered sensation?

C

A nurses assessment of a teenage girl reveals that her shoulders are not level and that she has one prominent scapula that is accentuated by bending forward. The nurse should expect to read about what health problem in the patients electronic health record? A) Lordosis B) Kyphosis C) Scoliosis D) Muscular dystrophy

C

A patient has been experiencing progressive increases in knee pain and diagnostic imaging reveals a worsening effusion in the synovial capsule. The nurse should anticipate which of the following? A) Arthrography B) Knee biopsy C) Arthrocentesis D) Electromyography

C

A patient injured in a motor vehicle accident has sustained a fracture to the diaphysis of the right femur. Of what is the diaphysis of the femur mainly constructed? A) Epiphyses B) Cartilage C) Cortical bone D) Cancellous bone

C

A patient presents at the dermatology clinic with suspected herpes simplex. The nurse knows to prepare what diagnostic test for this condition? A) Skin biopsy B) Patch test C) Tzanck smear D) Examination with a Woods light

C

The human body is designed to protect its vital parts. A fracture of what type of bone may interfere with the protection of vital organs? A) Long bones B) Short bones C) Flat bones D) Irregular bones

C

A nurse is caring for a patient whose cancer metastasis has resulted in bone pain. Which of the following are typical characteristics of bone pain? A) A dull, deep ache that is boring in nature B) Soreness or aching that may include cramping C) Sharp, piercing pain that is relieved by immobilization D) Spastic or sharp pain that radiates

A

The nurse in an ambulatory care center is admitting an older adult patient who has bright red moles on the skin. Benign changes in elderly skin that appear as bright red moles are termed what? A) Cherry angiomas B) Solar lentigo C) Seborrheickeratoses D) Xanthelasma

A

A nurse is caring for a patient whose skin cancer will soon be removed by excision. Which of the following actions should the nurse perform? A) Teach the patient about early signs of secondary blistering diseases. B) Teach the patient about self-care after treatment. C) Assess the patient's risk for recurrent malignancy. D) Assess the patient for adverse effects of radiotherapy.

28. A nurse is caring for a patient whose skin cancer will soon be removed by excision. Which of the following actions should the nurse perform? A) Teach the patient about early signs of secondary blistering diseases. B) Teach the patient about self-care after treatment. C) Assess the patient's risk for recurrent malignancy. D) Assess the patient for adverse effects of radiotherapy. Ans: B Feedback: Because many skin cancers are removed by excision, patients are usually treated in outpatient surgical units. The role of the nurse is to teach the patient about prevention of skin cancer and about self-care after treatment. Assessing the patient's risk for recurrent malignancy is primarily the role of the physician. Blistering diseases do not result from cancer or subsequent excision. Excision is not accompanied by radiotherapy.

A child is growing at a rate appropriate for his age. What cells are responsible for the secretion of bone matrix that eventually results in bone growth? A) Osteoblasts B) Osteocytes C) Osteoclasts D) Lamellae

A

A nurse is conducting a health interview and is assessing for integumentary conditions that are known to have a genetic component. What assessment question is most appropriate? A) Does anyone in your family have eczema or psoriasis? B) Have any of your family members been diagnosed with malignant melanoma? C) Do you have a family history of vitiligo or port-wine stains? D) Does any member of your family have a history of keloid scarring?

A

A nurse is providing care for a patient whose pattern of laboratory testing reveals longstanding hypocalcemia. What other laboratory result is most consistent with this finding? A) An elevated parathyroid hormone level B) An increased calcitonin level C) An elevated potassium level D) A decreased vitamin D level

A

A nurse is taking a health history on a patient with musculoskeletal dysfunction. What is the primary focus of this phase of the nurses assessment? A) Evaluating the effects of the musculoskeletal disorder on the patients function B) Evaluating the patients adherence to the existing treatment regimen C) Evaluating the presence of genetic risk factors for further musculoskeletal disorders D) Evaluating the patients active and passive range of motion

A

A nurse on the orthopedic unit is assessing a patients peroneal nerve. The nurse will perform this assessment by doing which of the following actions? A) Pricking the skin between the great and second toe B) Stroking the skin on the sole of the patients foot C) Pinching the skin between the thumb and index finger D) Stroking the distal fat pad of the small finger

A

A patient has just had an arthroscopy performed to assess a knee injury. What nursing intervention should the nurse implement following this procedure? A) Wrap the joint in a compression dressing. B) Perform passive range of motion exercises. C) Maintain the knee in flexion for up to 30 minutes. D) Apply heat to the knee.

A

A patient with human immunodeficiency virus (HIV) has sought care because of the recent development of new skin lesions. The nurse should interpret these lesions as most likely suggestive of what? A) A reduction in the patients CD4 count B) A reduction in the patients viral load C) An adverse effect of antiretroviral therapy D) Virus-induced changes in allergy status

A

A patients fracture is healing and callus is being deposited in the bone matrix. This process characterizes what phase of the bone healing process? A) The reparative phase B) The reactive phase C) The remodeling phase D) The revascularization phase

A

An 80-year-old patient is brought to the clinic by her son. The son asks the nurse why his mother has gotten so many spots on her skin. What would be an appropriate response by the nurse? A) As people age, they normally develop uneven pigmentation in their skin. B) These spots are called liver spots or age spots. C) Older skin is more apt to break down and tear, causing sores. D) These are usually the result of nutritional deficits earlier in life.

A

An older adult patient has symptoms of osteoporosis and is being assessed during her annual physical examination. The assessment shows that the patient will require further testing related to a possible exacerbation of her osteoporosis. The nurse should anticipate what diagnostic test? A) Bone densitometry B) Hip bone radiography C) Computed tomography (CT) D) Magnetic resonance imaging (MRI)

A

An older adult patient is diagnosed with a vitamin D deficiency. What would be an appropriate recommendation by the nurse? A) Spend time outdoors at least twice per week B) Increase intake of leafy green vegetables C) Start taking a multivitamin each morning D) Eat red meat at least once per week

A

An unresponsive Caucasian patient has been brought to the emergency room by EMS. While assessing this patient, the nurse notes that the patients face is a cherry-red color. What should the nurse suspect? A) Carbon monoxide poisoning B) Anemia C) Jaundice D) Uremia

A

Diagnostic tests show that a patients bone density has decreased over the past several years. The patient asks the nurse what factors contribute to bone density decreasing. What would be the nurses best response? A) For many people, lack of nutrition can cause a loss of bone density. B) Progressive loss of bone density is mostly related to your genes. C) Stress is known to have many unhealthy effects, including reduced bone density. D) Bone density decreases with age, but scientists are not exactly sure why this is the case.

A

The nurse is performing an assessment of a patients musculoskeletal system and is appraising the patients bone integrity. What action should the nurse perform during this phase of assessment? A) Compare parts of the body symmetrically. B) Assess extremities when in motion rather than at rest. C) Percuss as many joints as are accessible. D) Administer analgesia 30 to 60 minutes before assessment.

A

The outer layer of the epidermis provides the most effective barrier to penetration of the skin by environmental factors. Which of the following is an example of penetration by an environmental factor? A) An insect bite B) Dehydration C) Sunburn D) Excessive perspiration

A

While waiting to see the physician, a patient shows the nurse skin areas that are flat, nonpalpable, and have had a change of color. The nurse recognizes that the patient is demonstrating what? A) Macules B) Papules C) Vesicles D) Pustules

A

patient has been experiencing an unexplained decline in knee function and has consequently been scheduled for arthrography. The nurse should teach the patient about what process? A) Injection of a contrast agent into the knee joint prior to ROM exercises B) Aspiration of synovial fluid for serologic testing C) Injection of corticosteroids into the patients knee joint to facilitate ROM D) Replacement of the patients synovial fluid with a synthetic substitute

A

21. A patient with suspected adrenal insufficiency has been ordered an adrenocorticotropic hormone (ACTH) stimulation test. Administration of ACTH caused a marked increase in cortisol levels. How should the nurse interpret this finding? A) The patients pituitary function is compromised. B) The patients adrenal insufficiency is not treatable. C) The patient has insufficient hypothalamic function. D) The patient would benefit from surgery

A Feedback: An adrenal response to the administration of a stimulating hormone suggests inadequate production of the stimulating hormone. In this case, ACTH is produced by the pituitary and, consequently, pituitary hypofunction is suggested. Hypothalamic function is not relevant to the physiology of this problem. Treatment exists, although surgery is not likely indicated.

3. A patient with thyroid cancer has undergone surgery and a significant amount of parathyroid tissue has been removed. The nurse caring for the patient should prioritize what question when addressing potential complications? A) Do you feel any muscle twitches or spasms? B) Do you feel flushed or sweaty? C) Are you experiencing any dizziness or lightheadedness? D) Are you having any pain that seems to be radiating from your bones?

A Feedback: As the blood calcium level falls, hyperirritability of the nerves occurs, with spasms of the hands and feet and muscle twitching. This is characteristic of hypoparathyroidism. Flushing, diaphoresis, dizziness, and pain are atypical signs of the resulting hypocalcemia.

35. A patient with Cushing syndrome as a result of a pituitary tumor has been admitted for a transsphenoidal hypophysectomy. What would be most important for the nurse to monitor before, during, and after surgery? A) Blood glucose B) Assessment of urine for blood C) Weight D) Oral temperature

A Feedback: Before, during, and after this surgery, blood glucose monitoring and assessment of stools for blood are carried out. The patients blood sugar is more likely to be volatile than body weight or temperature. Hematuria is not a common complication.

28. Following an addisonian crisis, a patients adrenal function has been gradually regained. The nurse should ensure that the patient knows about the need for supplementary glucocorticoid therapy in which of the following circumstances? A) Episodes of high psychosocial stress B) Periods of dehydration C) Episodes of physical exertion D) Administration of a vaccine

A Feedback: During stressful procedures or significant illnesses, additional supplementary therapy with glucocorticoids is required to prevent addisonian crisis. Physical activity, dehydration and vaccine administration would not normally be sufficiently demanding such to require glucocorticoids.

23. A nurse works in a walk-in clinic. The nurse recognizes that certain patients are at higher risk for different disorders than other patients. What patient is at a greater risk for the development of hypothyroidism? A) A 75-year-old female patient with osteoporosis B) A 50-year-old male patient who is obese C) A 45-year-old female patient who used oral contraceptives D) A 25-year-old male patient who uses recreational drugs

A Feedback: Even though osteoporosis is not a risk factor for hypothyroidism, the condition occurs most frequently in older women.

1. The nurse is caring for a patient diagnosed with hypothyroidism secondary to Hashimotos thyroiditis. When assessing this patient, what sign or symptom would the nurse expect? A)Fatigue B) Bulging eyes C) Palpitations D) Flushed skin

A Feedback: Symptoms of hypothyroidism include extreme fatigue, hair loss, brittle nails, dry skin, voice huskiness or hoarseness, menstrual disturbance, and numbness and tingling of the fingers. Bulging eyes, palpitations, and flushed skin would be signs and symptoms of hyperthyroidism.

32. The nurse providing care for a patient with Cushing syndrome has identified the nursing diagnosis of risk for injury related to weakness. How should the nurse best reduce this risk? A) Establish falls prevention measures. B) Encourage bed rest whenever possible. C) Encourage the use of assistive devices. D) Provide constant supervision.

A Feedback: The nurse should take action to prevent the patients risk for falls. Bed rest carries too many harmful effects, however, and assistive devices may or may not be necessary. Constant supervision is not normally required or practicable.

10. You are developing a care plan for a patient with Cushing syndrome. What nursing diagnosis would have the highest priority in this care plan? A) Risk for injury related to weakness B) Ineffective breathing pattern related to muscle weakness C) Risk for loneliness related to disturbed body image D) Autonomic dysreflexia related to neurologic changes

A Feedback: The nursing priority is to decrease the risk of injury by establishing a protective environment. The patient who is weak may require assistance from the nurse in ambulating to prevent falls or bumping corners or furniture. The patients breathing will not be affected and autonomic dysreflexia is not a plausible risk. Loneliness may or may not be an issue for the patient, but safety is a priority.

13. A patient presents at the walk-in clinic complaining of diarrhea and vomiting. The patient has a documented history of adrenal insufficiency. Considering the patients history and current symptoms, the nurse should anticipate that the patient will be instructed to do which of the following? A) Increase his intake of sodium until the GI symptoms improve. B) Increase his intake of potassium until the GI symptoms improve. C) Increase his intake of glucose until the GI symptoms improve. D) Increase his intake of calcium until the GI symptoms improve.

A Feedback: The patient will need to supplement dietary intake with added salt during episodes of GI losses of fluid through vomiting and diarrhea to prevent the onset of addisonian crisis. While the patient may experience the loss of other electrolytes, the major concern is the replacement of lost sodium.

31. A patient is undergoing testing for suspected adrenocortical insufficiency. The care team should ensure that the patient has been assessed for the most common cause of adrenocortical insufficiency. What is the most common cause of this health problem? A) Therapeutic use of corticosteroids B) Pheochromocytoma C) Inadequate secretion of ACTH D) Adrenal tumor

A Feedback: Therapeutic use of corticosteroids is the most common cause of adrenocortical insufficiency. The other options also cause adrenocortical insufficiency, but they are not the most common causes.

A patient with thyroid cancer has undergone surgery and a significant amount of parathyroid tissue has been removed. The nurse caring for the patient should prioritize what question when addressing potential complications? A) "Do you feel any muscle twitches or spasms?" B) "Do you feel flushed or sweaty?" C) "Are you experiencing any dizziness or lightheadedness?" D) "Are you having any pain that seems to be radiating from your bones?"

A) "Do you feel any muscle twitches or spasms?"

A nurse works in a walk-in clinic. The nurse recognizes that certain patients are at higher risk for different disorders than other patients. What patient is at a greater risk for the development of hypothyroidism? A) A 75-year-old female patient with osteoporosis B) A 50-year-old male patient who is obese C) A 45-year-old female patient who used oral contraceptives D) A 25-year-old male patient who uses recreational drugs

A) A 75-year-old female patient with osteoporosis

A diabetic nurse is working for the summer at a camp for adolescents with diabetes. When providing information on the prevention and management of hypoglycemia, what action should the nurse promote? A) Always carry a form of fast-acting sugar. B) Perform exercise prior to eating whenever possible. C) Eat a meal or snack every 8 hours. D) Check blood sugar at least every 24 hours.

A) Always carry a form of fast-acting sugar.

A nurse is caring for a patient newly diagnosed with type 1 diabetes. The nurse is educating the patient about self-administration of insulin in the home setting. The nurse should teach the patient to do which of the following? A) Avoid using the same injection site more than once in 2 to 3 weeks. B) Avoid mixing more than one type of insulin in a syringe. C) Cleanse the injection site thoroughly with alcohol prior to injecting. D) Inject at a 45 angle.

A) Avoid using the same injection site more than once in 2 to 3 weeks.

A patient with Cushing's syndrome as a result of a pituitary tumor has been admitted for a transsphenoidal hypophysectomy. What would be most important for the nurse to monitor before, during, and after surgery? A) Blood glucose B) Assessment of urine for blood C) Weight D) Oral temperature

A) Blood glucose

A diabetes nurse educator is teaching a group of patients with type 1 diabetes about sick day rules. What guideline applies to periods of illness in a diabetic patient? A) Do not eliminate insulin when nauseated and vomiting. B) Report elevated glucose levels greater than 150 mg/dL. C) Eat three substantial meals a day, if possible. D) Reduce food intake and insulin doses in times of illness.

A) Do not eliminate insulin when nauseated and vomiting.

Following an addisonian crisis, a patient's adrenal function has been gradually regained. The nurse should ensure that the patient knows about the need for supplementary glucocorticoid therapy in which of the following circumstances? A) Episodes of high psychosocial stress B) Periods of dehydration C) Episodes of physical exertion D) Administration of a vaccine

A) Episodes of high psychosocial stress

The nurse is providing care for a patient with Chushing's syndrome has identified the nursing diagnosis of risk for injury r/t weakness. How should the nurse best reduce this risk? A) Establish falls prevention B) Encourage bed rest whenever possible C) Encourage the use of assistive devices D) Provide constant supervision

A) Establish falls prevention

A patient presents to the clinic complaining of symptoms that suggest diabetes. What criteria would support checking blood levels for the diagnosis of diabetes? A) Fasting plasma glucose greater than or equal to 126 mg/dL B) Random plasma glucose greater than 150 mg/dL C) Fasting plasma glucose greater than 116 mg/dL on 2 separate occasions D) Random plasma glucose greater than 126 mg/dL

A) Fasting plasma glucose greater than or equal to 126 mg/dL

The nurse is caring for a patient diagnosed with hypothyroidism secondary to Hashimoto's thyroiditis. When assessing this patient, what sign or symptom would the nurse expect? A) Fatigue B) Bulging eyes C) Palpitations D) Flushed skin

A) Fatigue

A patient with Cushing's syndrome has been hospitalized after a fall. The dietitian consulted works with the patient to improve the patient's nutritional intake. What foods should a patient with Cushing's syndrome eat to optimize health? Select all that apply. A) Foods high in vitamin D B) Foods high in calories C) Foods high in proteins D) Foods high in calcium E) Foods high in sodium

A) Foods high in vitamin D C) Foods high in proteins D) Foods high in calcium

A diabetic educator is discussing sick day rules with a newly diagnosed type 1 diabetic. The educator is aware that the patient will require further teaching when the patient states what? A) I will not take my insulin on the days when I am sick, but I will certainly check my blood sugar every 2 hours. B) If I cannot eat a meal, I will eat a soft food such as soup, gelatin, or pudding six to eight times a day. C) I will call the doctor if I am not able to keep liquids in my body due to vomiting or diarrhea. D) I will call the doctor if my blood sugar is over 300 mg/dL or if I have ketones in my urine.

A) I will not take my insulin on the days when I am sick, but I will certainly check my blood sugar every 2 hours.

A patient has been brought to the emergency department by paramedics after being found unconscious. The patients Medic Alert bracelet indicates that the patient has type 1 diabetes and the patients blood glucose is 22 mg/dL (1.2 mmol/L). The nurse should anticipate what intervention? A) IV administration of 50% dextrose in water B) Subcutaneous administration of 10 units of Humalog C) Subcutaneous administration of 12 to 15 units of regular insulin D) IV bolus of 5% dextrose in 0.45% NaCl

A) IV administration of 50% dextrose in water

A nurse is conducting a class on how to self-manage insulin regimens. A patient asks how long a vial of insulin can be stored at room temperature before it goes bad. What would be the nurses best answer? A) If you are going to use up the vial within 1 month it can be kept at room temperature. B) If a vial of insulin will be used up within 21 days, it may be kept at room temperature. C) If a vial of insulin will be used up within 2 weeks, it may be kept at room temperature. D) If a vial of insulin will be used up within 1 week, it may be kept at room temperature.

A) If you are going to use up the vial within 1 month it can be kept at room temperature.

A patient presents at the walk-in clinic complaining of diarrhea and vomiting. The patient has a documented history of adrenal insufficiency. Considering the patient's history and current symptoms, the nurse should anticipate that the patient will be instructed to do which of the following? A) Increase his intake of sodium until the GI symptoms improve. B) Increase his intake of potassium until the GI symptoms improve. C) Increase his intake of glucose until the GI symptoms improve. D) Increase his intake of calcium until the GI symptoms improve.

A) Increase his intake of sodium until the GI symptoms improve.

A patient with a longstanding diagnosis of type 1 diabetes has a history of poor glycemic control. The nurse recognizes the need to assess the patient for signs and symptoms of peripheral neuropathy. Peripheral neuropathy constitutes a risk for what nursing diagnosis? A) Infection B) Acute pain C) Acute confusion D) Impaired urinary elimination

A) Infection

A patient has been living with type 2 diabetes for several years, and the nurse realizes that the patient is likely to have minimal contact with the health care system. In order to ensure that the patient maintains adequate blood sugar control over the long term, the nurse should recommend which of the following? A) Participation in a support group for persons with diabetes B) Regular consultation of websites that address diabetes management C) Weekly telephone check-ins with an endocrinologist D) Participation in clinical trials relating to antihyperglycemics

A) Participation in a support group for persons with diabetes

You are developing a care plan for a patient with Cushing's syndrome. What nursing diagnosis would have the highest priority in this care plan? A) Risk for injury r/t weakness B) Ineffective breathing pattern r/t muscle weakneess C) Risk for loneliness r/t disturbed body image D) Autonomic dysreflexia r/t neurologic changes

A) Risk for injury r/t weakness

The most recent blood work of a patient with a longstanding diagnosis of type 1 diabetes has shown the presence of microalbuminuria. What is the nurses most appropriate action? A) Teach the patient about actions to slow the progression of nephropathy. B) Ensure that the patient receives a comprehensive assessment of liver function. C) Determine whether the patient has been using expired insulin. D) Administer a fluid challenge and have the test repeated.

A) Teach the patient about actions to slow the progression of nephropathy.

A patient with suspected adrenal insufficiency has been ordered an adrenocorticotropic hormone (ACTH) stimulation test. Administration of ACTH caused a marked increase in cortisol levels. How should the nurse interpret this finding? A) The patient's pituitary function is compromised B) The patient's adrenal insufficiency is not treatable C) The patient has insufficient hypothalamic function D) The patient would benefit from surgery

A) The patient's pituitary function is comprimised

A patient is undergoing testing for suspected adrenocortical insufficiency. The care team should ensure that the patient has been assessed for the most common cause of adrenocortical insufficiency. What is the most common cause of this health problem? A) Therapeutic use of corticosteroids B) Pheochromocytoma C) Inadequate secretion of ACTH D) Adrenal tumor

A) Therapeutic use of corticosteroids

A 15-year-old child is brought to the emergency department with symptoms of hyperglycemia and is subsequently diagnosed with diabetes. Based on the fact that the childs pancreatic beta cells are being destroyed, the patient would be diagnosed with what type of diabetes? A) Type 1 diabetes B) Type 2 diabetes C) Noninsulin-dependent diabetes D) Prediabetes

A) Type 1 diabetes

The nurse is performing a shift assessment of a patient with aldosteronism. What assessments should the nurse include? Select all that apply. A) Urine output B) Signs or symptoms of venous thromoembolism C) Peripheral pulses D) Blood pressure E) Skin integrity

A) Urine output D) Blood pressure

A nurse is explaining a patients decreasing bone density in terms of the balance between bone resorption and formation. What dietary nutrients and hormones play a role in the resorption and formation of adult bones? Select all that apply. A) Thyroid hormone B) Growth hormone C) Estrogen D) Vitamin B12 E) Luteinizing hormone

A, B, C

The nurse is assessing a patient for dietary factors that may influence her risk for osteoporosis. The nurse should question the patient about her intake of what nutrients? Select all that apply. A) Calcium B) Simple carbohydrates C) Vitamin D D) Protein E) Soluble fiber

A, C

33. A patient with Cushing syndrome has been hospitalized after a fall. The dietician consulted works with the patient to improve the patients nutritional intake. What foods should a patient with Cushing syndrome eat to optimize health? Select all that apply. A) Foods high in vitamin D B) Foods high in calories C) Foods high in protein D) Foods high in calcium E) Foods high in sodium

A, C, D Feedback: Foods high in vitamin D, protein, and calcium are recommended to minimize muscle wasting and osteoporosis. Referral to a dietitian may assist the patient in selecting appropriate foods that are also low in sodium and calories.

11. The nurse is performing a shift assessment of a patient with aldosteronism. What assessments should the nurse include? Select all that apply. A) Urine output B) Signs or symptoms of venous thromboembolism C) Peripheral pulses D) Blood pressure E) Skin integrity

A, D Feedback: The principal action of aldosterone is to conserve body sodium. Alterations in aldosterone levels consequently affect urine output and BP. The patients peripheral pulses, risk of VTE, and skin integrity are not typically affected by aldosteronism.

The policies and procedures on a preoperative unit are being amended to bring them closer into alignment with the focus of the Surgical Care Improvement Project (SCIP). What intervention most directly addresses the priorities of the SCIP? Actions aimed at increasing participation of families in planning care Actions aimed at preventing surgical site infections Actions aimed at increasing interdisciplinary collaboration Actions aimed at promoting the use of complementary and alternative medicine (CAM)

Actions aimed at preventing surgical site infections

A nurse is planning the care of a patient with herpes zoster. What medication, if administered within the first 24 hours of the initial eruption, can arrest herpes zoster A) Prednisone (Deltasone) B) Azanthioprine (Imuran) C) Triamcinolone (Kenalog) D) Acyclovir (Zovirax)

Acyclovir (Zovirax) Acyclovir, if started early, is effective in significantly reducing the pain and halting the progression of the disease.

The nurse is caring for a 78-year-old female patient who is scheduled for surgery to remove her brain tumor. The patient is very apprehensive and keeps asking when she will get her preoperative medicine. The medicine is ordered to be given on call to OR. When would be the best time to give this medication? As soon as possible, in order to alleviate the patients anxiety As the patient is transferred to the OR bed When the porter arrives on the floor to take the patient to surgery After being notified by the OR and before other preoperative preparations

After being notified by the OR and before other preoperative preparations

The nurse is performing a preoperative assessment on a patient going to surgery. The patient informs the nurse that he drinks approximately two bottles of wine each day and has for the last several years. What postoperative difficulties can the nurse anticipate for this patient? Alcohol withdrawal syndrome immediately following surgery Alcohol withdrawal syndrome 2 to 4 days after his last alcohol drink Alcohol withdrawal syndrome upon administration of general anesthesia Alcohol withdrawal syndrome 1 week after his last alcohol drink

Alcohol withdrawal syndrome 2 to 4 days after his last alcohol drink

A 55-year-old woman is scheduled to have a chemical face peel. The nurse is aware that the patient is likely seeking treatment for which of the following? A) Wrinkles near the lips and eyes B) Removal of acne scars C) Vascular lesions on the cheeks D) Real or perceived misshaping of the eyes

Ans: A Feedback: Chemical face peeling is especially useful for wrinkles at the upper and lower lip, forehead, and periorbital areas. Chemical face peeling does not remove acne scars, remove vascular lesions, or reshape the eyes.

A patient's blistering disorder has resulted in the formation of multiple lesions in the patient's mouth. What intervention should be included in the patient's plan of care? A) Provide chlorhexidine solution for rinsing the patient's mouth. B) Avoid providing regular mouth care until the patient's lesions heal. C) Liaise with the primary care provider to arrange for parenteral nutrition. D) Encourage the patient to gargle with a hypertonic solution after each meal.

Ans: A Feedback: Frequent rinsing of the mouth with chlorhexidine solution is prescribed to rid the mouth of debris and to soothe ulcerated areas. A hypertonic solution would be likely to cause pain and further skin disruption. Meticulous mouth care should be provided and there is no reason to provide nutrition parenterally.

21. A patient has just been diagnosed with psoriasis and frequently has lesions around his right eye. What should the nurse teach the patient about topical corticosteroid use on these lesions? A) Cataract development is possible. B) The ointment is likely to cause weeping. C) Corticosteroid use is contraindicated on these lesions. D) The patient may develop glaucoma.

Ans: A Feedback: Patients using topical corticosteroid preparations repeatedly on the face and around the eyes should be aware that cataract development is possible. Weeping and glaucoma are less likely. There is no consequent risk of glaucoma.

A patient presents at the free clinic with a black, wart-like lesion on his face, stating, "I've done some research, and I'm pretty sure I have malignant melanoma." Subsequent diagnostic testing results in a diagnosis of seborrheic keratosis. The nurse should recognize what significance of this diagnosis? A) The patient requires no treatment unless he finds the lesion to be cosmetically unacceptable. B) The patient's lesion will be closely observed for 6 months before a plan of treatment is chosen. C) The patient has one of the few dermatologic malignancies that respond to chemotherapy. D) The patient will likely require wide excision.

Ans: A Feedback: Seborrheic keratoses are benign, wart like lesions of various sizes and colors, ranging from light tan to black. There is no harm in allowing these growths to remain because there is no medical significance to their presence.

29. A public health nurse has reviewed local data about the incidence and prevalence of burn injuries in the community. These data are likely to support what health promotion effort? A) Education about home safety B) Education about safe storage of chemicals C) Education about workplace health threats D) Education about safe driving

Ans: A Feedback: A large majority of burns occur in the home setting; educational interventions should address this epidemiologic trend.

A patient has recently been diagnosed with advanced malignant melanoma and is scheduled for a wide excision of the tumor on her chest. In writing the plan of care for this patient, what major nursing diagnosis should the nurse include? A) Deficient Knowledge about Early Signs of Melanoma B) Chronic Pain Related to Surgical Excision and Grafting C) Depression Related to Reconstructive Surgery D) Anxiety Related to Lack of Social Support

Ans: A Feedback: The fact that the patient's disease was not reported until an advanced stage suggests that the patient lacked knowledge about skin lesions. Excision does not result in chronic pain. Reconstructive surgery is not a certainty, and will not necessarily lead to depression. Anxiety is likely, but this may or may not be related to a lack of social support.

A nurse is providing self-care education to a patient who has been receiving treatment for acne vulgaris. What instruction should the nurse provide to the patient? A) Wash your face with water and gentle soap each morning and evening. B) Before bedtime, clean your face with rubbing alcohol on a cotton pad. C) Gently burst new pimples before they form a visible 'head'. D) Set aside time each day to squeeze blackheads and remove the plug.

Ans: A Feedback: The nurse should inform the patient to wash the face and other affected areas with mild soap and water twice each day to remove surface oils and prevent obstruction of the oil glands. Cleansing with rubbing alcohol is not recommended and all forms of manipulation should be avoided.

A nurse is caring for a patient admitted to the medical unit with a diagnosis of pemphigus vulgaris. When writing the care plan for this patient, what nursing diagnoses should be included? A) Risk for Infection Related to Lesions B) Impaired Skin Integrity Related to Epidermal Blisters C) Disturbed Body Image Related to Presence of Skin Lesions D) Acute Pain Related to Disruption in Skin Integrity E) Hyperthermia Related to Disruptions in Thermoregulation

Ans: A, B, C, D Feedback: Blistering diseases disrupt skin integrity and are associated with pain and a risk for infection. Because of the visibility of blisters, body image is often affected. The patient faces a risk for hypothermia, not hyperthermia.

A nurse educator is teaching a group of medical nurses about Kaposi's sarcoma. What would the educator identify as characteristics of endemic Kaposi's sarcoma? Select All That Apply. A) Affects people predominantly in the eastern half of Africa B) Affects men more than women C) Does not affect children D) Cannot infiltrate E) Can progress to lymphadenopathic forms

Ans: A, B, E Feedback: Endemic (African) Kaposi's sarcoma affects people predominantly in the eastern half of Africa, near the equator. Men are affected more often than women, and children can be affected as well. The disease may resemble classic KS or it may infiltrate and progress to lymphadenopathic forms.

A patient has received a diagnosis of irritant contact dermatitis. What action should the nurse prioritize in the patient's subsequent care? A) Teaching the patient to safely and effectively administer immunosuppressants B) Helping the patient identify and avoid the offending agent C) Teaching the patient how to maintain meticulous skin hygiene D) Helping the patient perform wound care in the home environment

Ans: B Feedback: A focus of care for patients with irritant contact dermatitis is identifying and avoiding the offending agent. Immunosuppressants are not used to treat eczema and wound care is not normally required, except in cases of open lesions. Poor hygiene has no correlation with contact dermatitis.

A patient comes to the dermatology clinic requesting the removal of a port-wine stain on his right cheek. The nurse knows that the procedure especially useful in treating cutaneous vascular lesions such as port-wine stains is what? A) Skin graft B) Laser treatment C) Chemical face peeling D) Free flap

Ans: B Feedback: Argon lasers are useful in treating cutaneous vascular lesions such as port-wine stains. Skin grafts, chemical face peels, and free flaps would not be used to remove a port-wine stain.

A patient with a chronic diabetic wound is being discharged after receiving a skin graft to aid wound healing. What direction should the nurse include in home care instructions? A) Gently massage the graft site daily to promote perfusion. B) Protect the graft from direct sunlight and temperature extremes. C) Protect the graft site from any form of moisture for at least 12 weeks. D) Apply antibiotic ointment to the graft site and donor site daily.

Ans: B Feedback: Both the donor site and the grafted area must be protected from exposure to extremes in temperature, external trauma, and sunlight because these areas are sensitive, especially to thermal injuries. Antibiotic ointments are not typically prescribed and massage may damage these fragile sites. There is no need to protect the sites from all forms of moisture for the long term.

A school nurse has sent home four children who show evidence of pediculosis capitis. What is an important instruction the nurse should include in the note being sent home to parents? A) The child's scalp should be monitored for 48 to 72 hours before starting treatment. B) Nits may have to be manually removed from the child's hair shafts. C) The disease is self-limiting and symptoms will abate within 1 week. D) Efforts should be made to improve the child's level of hygiene.

Ans: B Feedback: Treatment for head lice should begin promptly and may require manual removal of nits following medicating shampoo. Head lice are not related to a lack of hygiene. Treatment is necessary because the condition will not likely resolve spontaneously within 1 week.

When caring for a patient with toxic epidermal necrolysis (TEN), the critical care nurse assesses frequently for high fever, tachycardia, and extreme weakness and fatigue. A) Possible malignancy B) Epidermal necrosis C) Neurologic involvement D) Increased metabolic needs E) Possible gastrointestinal mucosal sloughing

Ans: B, D, E Feedback: Assessment for high fever, tachycardia, and extreme weakness and fatigue is essential because these factors indicate the process of epidermal necrosis, increased metabolic needs, and possible gastrointestinal and respiratory mucosal sloughing. These factors are less likely to suggest malignancy or neurologic involvement, as these are not common complications of TEN.

An emergency department nurse is assessing a 17-year-old soccer player who presented with a knee injury. The patient's description of the injury indicates that his knee was struck medially while his foot was on the ground. The nurse knows that the patient likely has experienced what injury? A) Lateral collateral ligament injury B) Medial collateral ligament injury C) Anterior cruciate ligament injury D) Posterior cruciate ligament injury

Ans: A Feedback: When the knee is struck medially, damage may occur to the lateral collateral ligament. If the knee is struck laterally, damage may occur to the medial collateral ligament. The ACL and PCL are not typically injured in this way.

A nurse is caring for a patient who has a diagnosis of bullous pemphigoid and who is being treated on the medical unit. When providing hygiene for this patient, the nurse should perform which of the following actions? A) After washing, wipe lesions with sterile gauze to remove cellular debris. B) Apply antibiotic ointment to lesions after washing. C) Apply cornstarch to the patient's skin after bathing to facilitate mobility. D) Avoid using water to cleanse the patient's skin in order to maintain skin integrity.

Ans: C Feedback: After the patient's skin is bathed, it is dried carefully and dusted liberally with nonirritating powder (e.g., cornstarch), which enables the patient to move about freely in bed. Open blisters should not normally be wiped and antibiotics are not applied to wound beds in the absence of a secondary infection. Water can safely be used to provide hygiene.

A nurse is providing care for a patient who has psoriasis. The nurse is aware of the sequelae that can result from this health problem. Following the appearance of skin lesions, the nurse should prioritize A) Assessment of the patient's stool for evidence of intestinal sloughing B) Assessment of the patient's apical heart rate for dysrhythmias C) Assessment of the patient's joints for pain and decreased range of motion D) Assessment for cognitive changes resulting from neurologic lesions

Ans: C Feedback: Asymmetric rheumatoid factornegative arthritis of multiple joints occurs in up to 30% of people with psoriasis, most typically after the skin lesions appear. The most typical joints affected include those in the hands or feet, although sometimes larger joints such as the elbow, knees, or hips may be affected. As such, the nurse should assess for this musculoskeletal complication. GI, cardiovascular, and neurologic function are not affected by psoriasis.

An older adult resident of a long-term care facility has been experiencing generalized pruritus that has become more severe in recent weeks. What intervention should the nurse add to this resident's plan of care? A) Avoid the application of skin emollients. B) Apply antibiotic ointment as ordered following baths. C) Avoid using hot water during the patient's baths. D) Administer acetaminophen 4 times daily as ordered.

Ans: C Feedback: If baths have been prescribed, the patient is reminded to use tepid (not hot) water and to shake off the excess water and blot between intertriginous areas (body folds) with a towel. Skin emollients should be applied to reduce pruritus. Acetaminophen and antibiotics do not reduce pruritus.

A nurse is working with a family whose 5 year-old daughter has been diagnosed with impetigo. What educational intervention should the nurse include in this family's care? A) Ensuring that the family knows that impetigo is not contagious B) Teaching about the safe and effective use of topical corticosteroids C) Teaching about the importance of maintaining high standards of hygiene D) Ensuring that the family knows how to safely burst the child's vesicles

Ans: C Feedback: Impetigo is associated with unhygienic conditions; educational interventions to address this are appropriate. The disease is contagious, thus vesicles should not be manually burst. Because of the bacterial etiology, corticosteroids are ineffective.

A patient is admitted to the intensive care unit with what is thought to be toxic epidermal necrolysis (TEN). When assessing the health history of the patient, the nurse would be alert to what precipitating factor? A) Recent heavy ultraviolet exposure B) Substandard hygienic conditions C) Recent administration of new medications D) Recent varicella infection

Ans: C Feedback: In adults, TEN is usually triggered by a reaction to medications. Antibiotics, antiseizure agents, butazones, and sulfonamides are the most frequent medications implicated. TEN is unrelated to UV exposure, hygiene, or varicella infection.

A patient has a diagnosis of seborrhea and has been referred to the dermatology clinic, where the nurse contributes to care. When planning this patient's care, the nurse should include which of the following nursing diagnoses? A) Risk for Deficient Fluid Volume Related to Excess Sebum Synthesis B) Ineffective Thermoregulation Related to Occlusion of Sebaceous Glands C) Disturbed Body Image Related to Excess Sebum Production D) Ineffective Tissue Perfusion Related to Occlusion of Sebaceous Glands

Ans: C Feedback: Seborrhea causes highly visible manifestations that are likely to have a negative effect on the patient's body image. Seborrhea does not normally affect fluid balance, thermoregulation, or tissue perfusion.

A nurse is leading a health promotion workshop that is focusing on cancer prevention. What action is most likely to reduce participants' risks of basal cell carcinoma (BCC)? A) Teaching participants to improve their overall health through nutrition B) Encouraging participants to identify their family history of cancer C) Teaching participants to limit their sun exposure D) Teaching participants to control exposure to environmental and occupational radiation

Ans: C Feedback: Sun exposure is the best known and most common cause of BCC. BCC is not commonly linked to general health debilitation, family history, or radiation exposure.

37. A public health nurse is participating in a health promotion campaign that has the goal of improving outcomes related to skin cancer in the community. What action has the greatest potential to achieve this goal? A) Educating participants about the relationship between general health and the risk of skin cancer B) Educating participants about treatment options for skin cancer C) Educating participants about the early signs and symptoms of skin cancer D) Educating participants about the health risks associated with smoking and assisting with smoking cessation

Ans: C Feedback: The best hope of decreasing the incidence of skin cancer lies in educating patients about the early signs. There is a relationship between general health and skin cancer, but teaching individuals to identify the early signs and symptoms is more likely to benefit overall outcomes related to skin cancer. Teaching about treatment options is not likely to have a major effect on outcomes of the disease. Smoking is not among the major risk factors for skin cancer.

A patient has just been told that he has malignant melanoma. The nurse caring for this patient should anticipate that the patient will undergo what treatment? A) Chemotherapy B) Immunotherapy C) Wide excision D) Radiation therapy

Ans: C Feedback: Wide excision is the primary treatment for malignant melanoma, which removes the entire lesion and determines the level and staging. Chemotherapy may be used after the melanoma is excised. Immunotherapy is experimental and radiation therapy is palliative.

A patient with a suspected malignant melanoma is referred to the dermatology clinic. The nurse knows to facilitate what diagnostic test to rule out a skin malignancy? A) Tzanck smear B) Skin biopsy C) Patch testing D) Skin scrapings

B

A 65-year-old man presents at the clinic complaining of nodules on both legs. The man tells the nurse that his son, who is in medical school, encouraged him to seek prompt care and told him that the nodules are related to the fact that he is Jewish.What health problem should the nurse suspect? A) Stasis ulcers B) Bullous pemphigoid C) Psoriasis D) Classic Kaposi's sarcoma

Ans: D Feedback: Classic Kaposi's sarcoma occurs predominantly in men of Mediterranean or Jewish ancestry between 40 and 70 years of age. Most patients have nodules or plaques on the lower extremities that rarely metastasize beyond this area. Classic KS is chronic, relatively benign, and rarely fatal. Stasis ulcers do not create nodules. Bullous pemphigoid is characterized by blistering. Psoriasis characteristically presents with silvery plaques.

A nurse is assessing a teenage patient with acne vulgaris. The patient's mother states, "I keep telling him that this is what happens when you eat as much chocolate as he does."What aspect of the pathophysiology of acne should inform the nurse's response? A) A sudden change in patient's diet may exacerbate, rather than alleviate, the patient's symptoms. B) Chocolate is not among the foods that are known to cause acne. C) Elimination of chocolate from the patient's diet will likely lead to resolution within several months. D) Diet is thought to play a minimal role in the development of acne.

Ans: D Feedback: Diet is not believed to play a major role in acne therapy. A change in diet is not known to exacerbate symptoms.

8. A nurse is providing care for a patient who has developed Kaposi's sarcoma secondary to HIV infection. The nurse should be aware that this form of malignancy originates in what part of the body? A) Connective tissue cells in diffuse locations B) Smooth muscle cells of the gastrointestinal and respiratory tract C) Neural tissue of the brain and spinal cord D) Endothelial cells lining small blood vessels

Ans: D Feedback: Kaposi's sarcoma (KS) is a malignancy of endothelial cells that line the small blood vessels. It does not originate in connective tissue, smooth muscle cell

A patient has just undergone surgery for malignant melanoma. Which of the following nursing actions should be prioritized? A) Maintain the patient on bed rest for the first 24 hours postoperative. B) Apply distraction techniques to relieve pain. C) Provide soft or liquid diet that is high in protein to assist with healing. D) Anticipate the need for, and administer, appropriate analgesic medications.

Ans: D Feedback: Nursing interventions after surgery for a malignant melanoma center on promoting comfort, because wide excision surgery may be necessary. Anticipating the need for and administering appropriate analgesic medications are important. Distraction techniques may be appropriate for some patients, but these are not a substitute for analgesia. Bed rest and a modified diet are not necessary.

A 30-year-old male patient has just returned from the operating room after having a "flap" done following a motorcycle accident. The patient's wife asks the nurse about the major complications following this type of surgery. What would be the nurse's best response? A) The major complication is when the patient develops chronic pain. B) The major complication is when the patient loses sensation in the flap. C) The major complication is when the pedicle tears loose and the flap dies. D) The major complication is when the blood supply fails and the tissue in the flap dies.

Ans: D Feedback: The major complication of a flap is necrosis of the pedicle or base as a result of failure of the blood supply. This is more likely than tearing of the pedicle and chronic pain and is more serious than loss of sensation.

A nurse is caring for a patient whose chemical injury has necessitated a skin graft to his left hand. The nurse enters the room and observes that the patient is performing active ROM exercises with the affected hand. How should the nurse best respond? A) Liaise with the physical therapist to ensure that the patient is performing exercises safely. B) Validate the patient's efforts to increase blood perfusion to the graft site. C) Remind the patient that ROM exercises should be passive, not active. D) Remind the patient of the need to immobilize the graft to facilitate healing.

Ans: D Feedback: The nurse should instruct the patient to keep the affected part immobilized as much as possible in order to facilitate healing. Passive ROM exercises can be equally as damaging as active ROM.

A patient who has sustained third-degree facial burns and a facial fracture is undergoing reconstructive surgery and implantation of a prosthesis. The nurse has identified a nursing diagnosis of Disturbed Body Image Related to Disfigurement. What would be an appropriate nursing intervention related to this diagnosis? A) Referring the patient to a speech therapist B) Gradually adding soft foods to diet C) Administering analgesics as prescribed D) Teaching the patient how to use and care for the prosthesis

Ans: D Feedback: The process of facial reconstruction is often slow and tedious. Because a person's facial appearance affects self-esteem so greatly, this type of reconstruction is often a very emotional experience for the patient. Reinforcement of the patient's successful coping strategies improves self-esteem. If prosthetic devices are used, the patient is taught how to use and care for them to gain a sense of greater independence. This is an intervention that relates to Disturbed Body Image in these patients. None of the other listed interventions relates directly to the diagnosis of Disturbed Body Image.

A patient who has undergone a lower limb amputation is preparing to be discharged home. What outcome is necessary prior to discharge? A) Patient can demonstrate safe use of assistive devices. B) Patient has a healed, nontender, nonadherent scar. C) Patient can perform activities of daily living independently. D) Patientis free of pain.

Ans: A Feedback: A patient should be able to use assistive devices appropriately and safely prior to discharge. Scar formation will not be complete at the time of hospital discharge. It is anticipated that the patient will require some assistance with ADLs postdischarge. Pain should be well managed, but may or may not be wholly absent.

A patient was fitted with an arm cast after fracturing her humerus. Twelve hours after the application of the cast, the patient tells the nurse that her arm hurts. Analgesics do not relieve the pain. What would be the most appropriate nursing action? A) Prepare the patient for opening or bivalving of the cast. B) Obtain an order for a different analgesic. C) Encourage the patient to wiggle and move the fingers. D) Petal the edges of the patient's cast.

Ans: A Feedback: Acute compartment syndrome involves a sudden and severe decrease in blood flow to the tissues distal to an area of injury that results in ischemic necrosis if prompt, decisive intervention does not occur. Removing or bivalving the cast is necessary to relieve pressure. Ordering different analgesics does not address the underlying problem. Encouraging the patient to move the fingers or perform range-of-motion exercises will not treat or prevent compartment syndrome. Petaling the edges of a cast with tape prevents abrasions and skin breakdown, not compartment syndrome.

A rehabilitation nurse is working with a patient who has had a below-the-knee amputation. The nurse knows the importance of the patient's active participation in self-care. In order to determine the patient's ability to be an active participant in self-care, the nurse should prioritize assessment of what variable? A) The patient's attitude B) The patient's learning style C) The patient's nutritional status D) The patient's presurgical level of function

Ans: A Feedback: Amputation of an extremity affects the patient's ability to provide adequate self-care. The patient is encouraged to be an active participant in self-care. The patient and the nurse need to maintain positive attitudes and to minimize fatigue and frustration during the learning process. Balanced nutrition and the patient's learning style are important variables in the rehabilitation process but the patient's attitude is among the most salient variables. The patient's presurgical level of function may or may not affect participation in rehabilitation.

The surgical nurse is admitting a patient from postanesthetic recovery following the patient's below-the-knee amputation. The nurse recognizes the patient's high risk for postoperative hemorrhage and should keep which of the following at the bedside? A) A tourniquet B) A syringe preloaded with vitamin K C) A unit of packed red blood cells, placed on ice D) A dose of protamine sulfate

Ans: A Feedback: Immediate postoperative bleeding may develop slowly or may take the form of massive hemorrhage resulting from a loosened suture. A large tourniquet should be in plain sight at the patient's bedside so that, if severe bleeding occurs, it can be applied to the residual limb to control the hemorrhage. PRBCs cannot be kept at the bedside. Vitamin K and protamine sulfate are antidotes to warfarin and heparin, but are not administered to treat active postsurgical bleeding.

A 25-year-old man is involved in a motorcycle accident and injures his arm. The physician diagnoses the man with an intra-articular fracture and splints the injury. The nurse implements the teaching plan developed for this patient. What sequela of intra-articular fractures should the nurse describe regarding this patient? A) Post-traumatic arthritis B) Fat embolism syndrome (FES) C) Osteomyelitis D) Compartment syndrome

Ans: A Feedback: Intra-articular fractures often lead to post-traumatic arthritis. Research does not indicate a correlation between intra-articular fractures and FES, osteomyelitis, or compartment syndrome.

The patient scheduled for a Syme amputation is concerned about the ability to eventually stand on the amputated extremity. How should the nurse best respond to the patient's concern? A) "You will eventually be able to withstand full weight-bearing after the amputation." B) "You will have minimal weight-bearing on this extremity but you'll be taught how to use an assistive device." C) "You likely will not be able to use this extremity but you will receive teaching on use of a wheelchair." D) "You will be fitted for a prosthesis which may or may not allow you to walk."

Ans: A Feedback: Syme amputation (modified ankle disarticulation amputation) is performed most frequently for extensive foot trauma and produces a painless, durable extremity end that can withstand full weight-bearing. Therefore, each of the other teaching statements is incorrect.

A nurse is caring for a patient who had a right below-the-knee amputation (BKA). The nurse recognizes the importance of implementing measures that focus on preventing flexion contracture of the hip and maintaining proper positioning. Which of the following measures will best achieve these goals? A) Encouraging the patient to turn from side to side and to assume a prone position B) Initiating ROM exercises of the hip and knee 10 to 12 weeks after the amputation C) Minimizing movement of the flexor muscles of the hip D) Encouraging the patient to sit in a chair for at least 8 hours a day

Ans: A Feedback: The nurse encourages the patient to turn from side to side and to assume a prone position, if possible, to stretch the flexor muscles and to prevent flexion contracture of the hip. Postoperative ROM exercises are started early, because contracture deformities develop rapidly. ROM exercises include hip and knee exercises for patients with BKAs. The nurse also discourages sitting for prolonged periods of time.

A patient has returned to the postsurgical unit from the PACU after an above-the-knee amputation of the right leg. Results of the nurse's initial postsurgical assessment were unremarkable but the patient has called out. The nurse enters the room and observes copious quantities of blood at the surgical site. What should be the nurse's initial action? A) Apply a tourniquet. B) Elevate the residual limb. C) Apply sterile gauze. D) Call the surgeon.

Ans: A Feedback: The nurse should apply a tourniquet in the event of postsurgical hemorrhage. Elevating the limb and applying sterile gauze are likely insufficient to stop the hemorrhage. The nurse should attempt to control the immediate bleeding before contacting the surgeon.

A nurse is writing a care plan for a patient admitted to the emergency department (ED) with an open fracture. The nurse will assign priority to what nursing diagnosis for a patient with an open fracture of the radius? A) Risk for Infection B) Risk for Ineffective Role Performance C) Risk for Perioperative Positioning Injury D) Risk for Powerlessness

Ans: A Feedback: The patient has a significant risk for osteomyelitis and tetanus due to the fact that the fracture is open. Powerlessness and ineffective role performance are psychosocial diagnoses that may or may not apply, and which would be superseded by immediate physiologic threats such as infection. Surgical positioning injury is not plausible, since surgery is not likely indicated.

A public health nurse is organizing a campaign that will address the leading cause of musculoskeletal- related disability in the United States. The nurse should focus on what health problem? A) Osteoporosis B) Arthritis C) Hip fractures D) Lower back pain

B

7. A nurse on a burn unit is caring for a patient in the acute phase of burn care. While performing an assessment during this phase of burn care, the nurse recognizes that airway obstruction related to upper airway edema may occur up to how long after the burn injury? A) 2 days B) 3 days C) 5 days D) 1 week

Ans: A Feedback: Airway obstruction caused by upper airway edema can take as long as 48 hours to develop. Changes detected by x-ray and arterial blood gases may occur as the effects of resuscitative fluid and the chemical reaction of smoke ingredients with lung tissues become apparent.

18. A nurse is caring for a patient in the emergent/resuscitative phase of burn injury. During this phase, the nurse should monitor for evidence of what alteration in laboratory values? A) Sodium deficit B) Decreased prothrombin time (PT) C) Potassium deficit D) Decreased hematocrit

Ans: A Feedback: Anticipated fluid and electrolyte changes that occur during the emergent/resuscitative phase of burn injury include sodium deficit, potassium excess, base-bicarbonate deficit, and elevated hematocrit. PT does not typically decrease.

27. A patient is admitted to the burn unit after being transported from a facility 1000 miles away. The patient has burns to the groin area and circumferential burns to both upper thighs. When assessing the patients legs distal to the wound site, the nurse should be cognizant of the risk of what complication? A) Ischemia B) Referred pain C) Cellulitis D) Venous thromboembolism (VTE)

Ans: A Feedback: As edema increases, pressure on small blood vessels and nerves in the distal extremities causes an obstruction of blood flow and consequent ischemia. This complication is similar to compartment syndrome. Referred pain, cellulitis, and VTE are not noted complications that occur distal to the injury site.

35. A burn patient is transitioning from the acute phase of the injury to the rehabilitation phase. The patient tells the nurse, I cant wait to have surgery to reconstruct my face so I look normal again. What would be the nurses best response? A) That's something that you and your doctor will likely talk about after your scars mature. B) That is something for you to talk to your doctor about because it's not a nursing responsibility. C) I know this is really important to you, but you have to realize that no one can make you look like you used to. D) Unfortunately, it's likely that you will have most of these scars for the rest of your life.

Ans: A Feedback: Burn reconstruction is a treatment option after all scars have matured and is discussed within the first few years after injury. Even though this is not a nursing responsibility, the nurse should still respond appropriately to the patients query. It is true that the patient will not realistically look like he or she used to, but this does not instill hope.

9. A patient has been admitted to a burn intensive care unit with extensive full-thickness burns over 25% of the body. After ensuring cardiopulmonary stability, what would be the nurses immediate, priority concern when planning this patients care? A) Fluid status B) Risk of infection C) Nutritional status D) Psychosocial coping

Ans: A Feedback: During the early phase of burn care, the nurse is most concerned with fluid resuscitation, to correct large-volume fluid loss through the damaged skin. Infection control and early nutritional support are important, but fluid resuscitation is an immediate priority. Coping is a higher priority later in the recovery period.

3. A patient in the emergent/resuscitative phase of a burn injury has had blood work and arterial blood gases drawn. Upon analysis of the patients laboratory studies, the nurse will expect the results to indicate what? A) Hyperkalemia, hyponatremia, elevated hematocrit, and metabolic acidosis B) Hypokalemia, hypernatremia, decreased hematocrit, and metabolic acidosis C) Hyperkalemia, hypernatremia, decreased hematocrit, and metabolic alkalosis D) Hypokalemia, hyponatremia, elevated hematocrit, and metabolic alkalosis

Ans: A Feedback: Fluid and electrolyte changes in the emergent/resuscitative phase of a burn injury include hyperkalemia related to the release of potassium into the extracellular fluid, hyponatremia from large amounts of sodium lost in trapped edema fluid, hemoconcentration that leads to an increased hematocrit, and loss of bicarbonate ions that results in metabolic acidosis.

8. A patient has sustained a severe burn injury and is thought to have an impaired intestinal mucosal barrier. Since this patient is considered at an increased risk for infection, what intervention will best assist in avoiding increased intestinal permeability and prevent early endotoxin translocation? A) Early enteral feeding B) Administration of prophylactic antibiotics C) Bowel cleansing procedures D) Administration of stool softeners

Ans: A Feedback: If the intestinal mucosa receives some type of protection against permeability change, infection could be avoided. Early enteral feeding is one step to help avoid this increased intestinal permeability and prevent early endotoxin translocation. Antibiotics are seldom prescribed prophylactically because of the risk of promoting resistant strains of bacteria. A bowel cleansing procedure would not be ordered for this patient. The administration of stool softeners would not assist in avoiding increased intestinal permeability and prevent early endotoxin translocation.

25. An emergency department nurse has just received a patient with burn injuries brought in by ambulance. The paramedics have started a large-bore IV and covered the burn in cool towels. The burn is estimated as covering 24% of the patients body. How should the nurse best address the pathophysiologic changes resulting from major burns during the initial burn-shock period? A) Administer IV fluids B) Administer broad-spectrum antibiotics C) Administer IV potassium chloride D) Administer packed red blood cells

Ans: A Feedback: Pathophysiologic changes resulting from major burns during the initial burn-shock period include massive fluid losses. Addressing these losses is a major priority in the initial phase of treatment. Antibiotics and PRBCs are not normally administered. Potassium chloride would exacerbate the patients hyperkalemia.

12. An emergency department nurse learns from the paramedics that they are transporting a patient who has suffered injury from a scald from a hot kettle. What variables will the nurse consider when determining the depth of burn? A) The causative agent B) The patients preinjury health status C) The patients prognosis for recovery D) The circumstances of the accident

Ans: A Feedback: The following factors are considered in determining the depth of a burn: how the injury occurred, causative agent (such as flame or scalding liquid), temperature of the burning agent, duration of contact with the agent, and thickness of the skin. The patients preinjury status, circumstances of the accident, and prognosis for recovery are important, but are not considered when determining the depth of the burn.

36. A patient who is in the acute phase of recovery from a burn injury has yet to experience adequate pain control. What pain management strategy is most likely to meet this patients needs? A) A patient-controlled analgesia (PCA) system B) Oral opioids supplemented by NSAIDs C) Distraction and relaxation techniques supplemented by NSAIDs D) A combination of benzodiazepines and topical anesthetics

Ans: A Feedback: The goal of treatment is to provide a long-acting analgesic that will provide even coverage for this longterm discomfort. It is helpful to use escalating doses when initiating the medication to reach the level of pain control that is acceptable to the patient. The use of patient-controlled analgesia (PCA) gives control to the patient and achieves this goal. Patients cannot normally achieve adequate pain control without the use of opioids, and parenteral administration is usually required.

23. A patient is brought to the emergency department with a burn injury. The nurse knows that the first systemic event after a major burn injury is what? A) Hemodynamic instability B) Gastrointestinal hypermotility C) Respiratory arrest D) Hypokalemia

Ans: A Feedback: The initial systemic event after a major burn injury is hemodynamic instability, which results from loss of capillary integrity and a subsequent shift of fluid, sodium, and protein from the intravascular space into the interstitial spaces. This precedes GI changes. Respiratory arrest may or may not occur, largely depending on the presence or absence of smoke inhalation. Hypokalemia does not take place in the initial phase of recovery.

22. A nurse has reported for a shift at a busy burns and plastics unit in a large university hospital. Which patient is most likely to have life-threatening complications? A) A 4-year-old scald victim burned over 24% of the body B) A 27-year-old male burned over 36% of his body in a car accident C) A 39-year-old female patient burned over 18% of her body D) A 60-year-old male burned over 16% of his body in a brush fire

Ans: A Feedback: Young children and the elderly continue to have increased morbidity and mortality when compared to other age groups with similar injuries and present a challenge for burn care. This is an important factor when determining the severity of injury and possible outcome for the patient.

A nurse is planning the care of an older adult patient who will soon be discharged home after treatment for a fractured hip. In an effort to prevent future fractures, the nurse should encourage which of the following? Select all that apply. A) Regular bone density testing B) A high-calcium diet C) Use of falls prevention precautions D) Use of corticosteroids as ordered E) Weight-bearing exercise

Ans: A, B, C, E Feedback: Health promotion measures after an older adult's hip fracture include weight-bearing exercise, promotion of a healthy diet, falls prevention, and bone density testing. Corticosteroids have the potential to reduce bone density and increase the risk for fractures.

An older adult patient experienced a fall and required treatment for a fractured hip on the orthopedic unit. Which of the following are contributory factors to the incidence of falls and fractured hips among the older adult population? Select all that apply. A) Loss of visual acuity B) Adverse medication effects C) Slowed reflexes D) Hearing loss E) Muscle weakness

Ans: A, B, C, E Feedback: Older adults are generally vulnerable to falls and have a high incidence of hip fracture. Weak quadriceps muscles, medication effects, vision loss, and slowed reflexes are among the factors that contribute to the incidence of falls. Decreased hearing is not noted to contribute to the incidence of falls.

33. A patient is in the acute phase of a burn injury. One of the nursing diagnoses in the plan of care is Ineffective Coping Related to Trauma of Burn Injury. What interventions appropriately address this diagnosis? Select all that apply. A) Promote truthful communication. B) Avoid asking the patient to make decisions. C) Teach the patient coping strategies. D) Administer benzodiazepines as ordered. E) Provide positive reinforcement.

Ans: A, C, E Feedback: The nurse can assist the patient to develop effective coping strategies by setting specific expectations for behavior, promoting truthful communication to build trust, helping the patient practice appropriate strategies, and giving positive reinforcement when appropriate. The patient may benefit from being able to make decisions regarding his or her care. Benzodiazepines may be needed for short-term management of anxiety, but they are not used to enhance coping.

A nurse admits a patient who has a fracture of the nose that has resulted in a skin tear and involvement of the mucous membranes of the nasal passages. The orthopedic nurse is aware that this description likely indicates which type of fracture? A) Compression B) Compound C) Impacted D) Transverse

Ans: B Feedback: A compound fracture involves damage to the skin or mucous membranes and is also called an open fracture. A compression fracture involves compression of bone and is seen in vertebral fractures. An impacted fracture occurs when a bone fragment is driven into another bone fragment. A transverse fracture occurs straight across the bone shaft.

An older adult patient has fallen in her home and is brought to the emergency department by ambulance with a suspected fractured hip. X-rays confirm a fracture of the left femoral neck. When planning assessments during the patient's presurgical care, the nurse should be aware of the patient's heightened risk of what complication? A) Osteomyelitis B) Avascular necrosis C) Phantom pain D) Septicemia

Ans: B Feedback: Fractures of the neck of the femur may damage the vascular system that supplies blood to the head and the neck of the femur, and the bone may become ischemic. For this reason, AVN is common in patients with femoral neck fractures. Infections are not immediate complications and phantom pain applies to patients with amputations, not hip fractures.

4. A patient has experienced an electrical burn and has developed thick eschar over the burn site. Which of the following topical antibacterial agents will the nurse expect the physician to order for the wound? A) Silver sulfadiazine 1% (Silvadene) water-soluble cream B) Mafenide acetate 10% (Sulfamylon) hydrophilic-based cream C) Silver nitrate 0.5% aqueous solution D) Acticoat

Ans: B Feedback: Mafenide acetate 10% hydrophilic-based cream is the agent of choice when there is a need to penetrate thick eschar. Silver products do not penetrate eschar; Acticoat is a type of silver dressing.

An elite high school football player has been diagnosed with a shoulder dislocation. The patient has been treated and is eager to resume his role on his team, stating that he is not experiencing pain. What should the nurse emphasize during health education? A) The need to take analgesia regardless of the short-term absence of pain B) The importance of adhering to the prescribed treatment and rehabilitation regimen C) The fact that he has a permanently increased risk of future shoulder dislocations D) The importance of monitoring for intracapsular bleeding once he resumes playing

Ans: B Feedback: Patients who have experienced sports-related injuries are often highly motivated to return to their previous level of activity. Adherence to restriction of activities and gradual resumption of activities needs to be reinforced. Appropriate analgesia use must be encouraged, but analgesia does not necessarily have to be taken in the absence of pain. If healing is complete, the patient does not likely have a greatly increased risk of reinjury. Dislocations rarely cause bleeding after the healing process.

A nurse is caring for a patient who has suffered an unstable thoracolumbar fracture. Which of the following is the priority during nursing care? A) Preventing infection B) Maintaining spinal alignment C) Maximizing function D) Preventing increased intracranial pressure

Ans: B Feedback: Patients with an unstable fracture must have their spine in alignment at all times in order to prevent neurologic damage. This is a greater threat, and higher priority, than promoting function and preventing infection, even though these are both valid considerations. Increased ICP is not a high risk.

Which of the following is the most appropriate nursing intervention to facilitate healing in a patient who has suffered a hip fracture? A) Administer analgesics as required. B) Place a pillow between the patient's legs when turning. C) Maintain prone positioning at all times. D) Encourage internal and external rotation of the affected leg.

Ans: B Feedback: Placing a pillow between the patient's legs when turning prevents adduction and supports the patient's legs. Administering analgesics addresses pain but does not directly protect bone remodeling and promote healing. Rotation of the affected leg can cause dislocation and must be avoided. Prone positioning does not need to be maintained at all times.

The orthopedic nurse should assess for signs and symptoms of Volkmann's contracture if a patient has fractured which of the following bones? A) Femur B) Humerus C) Radial head D) Clavicle

Ans: B Feedback: The most serious complication of a supracondylar fracture of the humerus is Volkmann's ischemic contracture, which results from antecubital swelling or damage to the brachial artery. This complication is specific to humeral fractures.

A patient has sustained a long bone fracture and the nurse is preparing the patient's care plan. Which of the following should the nurse include in the care plan? A) Administer vitamin D and calcium supplements as ordered. B) Monitor temperature and pulses of the affected extremity. C) Perform passive range of motion exercises as tolerated. D) Administer corticosteroids as ordered.

Ans: B Feedback: The nurse should include monitoring for sufficient blood supply by assessing the color, temperature, and pulses of the affected extremity. Weight-bearing exercises are encouraged, but passive ROM exercises have the potential to cause pain and inhibit healing. Corticosteroids, vitamin D, and calcium are not normally administered.

A patient with a simple arm fracture is receiving discharge education from the nurse. What would the nurse instruct the patient to do? A) Elevate the affected extremity to shoulder level when at rest. B) Engage in exercises that strengthen the unaffected muscles. C) Apply topical anesthetics to accessible skin surfaces as needed. D) Avoid using analgesics so that further damage is not masked.

Ans: B Feedback: The nurse will encourage the patient to engage in exercises that strengthen the unaffected muscles. Comfort measures may include appropriate use of analgesics and elevation of the affected extremity to the heart level. Topical anesthetics are not typically used.

The nurse is providing care for a patient who has had a below-the-knee amputation. The nurse enters the patient's room and finds him resting in bed with his residual limb supported on pillow. What is the nurse's most appropriate action? A) Inform the surgeon of this finding. B) Explain the risks of flexion contracture to the patient. C) Transfer the patient to a sitting position. D) Encourage the patient to perform active ROM exercises with the residual limb.

Ans: B Feedback: The residual limb should not be placed on a pillow, because a flexion contracture of the hip may result. There is no acute need to contact the patient's surgeon. Encouraging exercise or transferring the patient does not address the risk of flexion contracture.

A nurse's assessment of a patient's knee reveals edema, tenderness, muscle spasms, and ecchymosis. The patient states that 2 days ago he ran 10 miles and now it "really hurts to stand up." The nurse should plan care based on the belief that the patient has experienced what? A) A first-degree strain B) A second-degree strain C) A first-degree sprain D) A second-degree sprain

Ans: B Feedback: A second-degree strain involves tearing of muscle fibers and is manifested by notable loss of load-bearing strength with accompanying edema, tenderness, muscle spasm, and ecchymosis. A first-degree strain reflects tearing of a few muscle fibers and is accompanied by minor edema, tenderness, and mild muscle spasm, without noticeable loss of function. However, this patient states a loss of function. A sprain normally involves twisting, which is inconsistent with the patient's overuse injury.

20. A patients burns have required a homograft. During the nurses most recent assessment, the nurse observes that the graft is newly covered with purulent exudate. What is the nurses most appropriate response? A) Perform mechanical dbridement to remove the exudate and prevent further infection. B) Inform the primary care provider promptly because the graft may need to be removed. C) Perform range of motion exercises to increase perfusion to the graft site and facilitate healing. D) Document this finding as an expected phase of graft healing.

Ans: B Feedback: An infected graft may need to be removed, thus the care provider should be promptly informed. ROM exercises will not resolve this problem and the nurse would not independently perform dbridement.

10. The nurse is preparing the patient for mechanical debridement and informs the patient that this will involve which of the following procedures? A) A spontaneous separation of dead tissue from the viable tissue B) Removal of eschar until the point of pain and bleeding occurs C) Shaving of burned skin layers until bleeding, viable tissue is revealed D) Early closure of the wound

Ans: B Feedback: Mechanical debridement can be achieved through the use of surgical scissors, scalpels, or forceps to remove the eschar until the point of pain and bleeding occurs. Mechanical debridement can also be accomplished through the use of topical enzymatic debridement agents. The spontaneous separation of dead tissue from the viable tissue is an example of natural debridement. Shaving the burned skin layers and early wound closure are examples of surgical debridement.

34. A patient who was burned in a workplace accident has completed the acute phase of treatment and the plan of care has been altered to prioritize rehabilitation. What nursing action should be prioritized during this phase of treatment? A) Monitoring fluid and electrolyte imbalances B) Providing education to the patient and family C) Treating infection D) Promoting thermoregulation

Ans: B Feedback: Patient and family education is a priority during rehabilitation. There should be no fluid and electrolyte imbalances in the rehabilitation phase. The presence of impaired thermoregulation or infection would suggest that the patient is still in the acute phase of burn recovery.

21. A nurse who is taking care of a patient with burns is asked by a family member why the patient is losing so much weight. The patient is currently in the intermediate phase of recovery. What would be the nurses most appropriate response to the family member? A) Hes on a calorie-restricted diet in order to divert energy to wound healing. B) His body has consumed his fat deposits for fuel because his calorie intake is lower than normal. C) He actually hasnt lost weight. Instead, theres been a change in the distribution of his body fat. D) He lost many fluids while he was being treated in the emergency phase of burn care.

Ans: B Feedback: Patients lose a great deal of weight during recovery from severe burns. Reserve fat deposits are catabolized as a result of hypermetabolism. Patients are not placed on a calorie restriction during recovery and fluid losses would not account for weight loss later in the recovery period. Changes in the overall distribution of body fat do not occur.

39. A home care nurse is performing a visit to a patients home to perform wound care following the patients hospital treatment for severe burns. While interacting with the patient, the nurse should assess for evidence of what complication? A) Psychosis B) Post-traumatic stress disorder C) Delirium D) Vascular dementia

Ans: B Feedback: Post-traumatic stress disorder (PTSD) is the most common psychiatric disorder in burn survivors, with a prevalence that may be as high as 45%. As a result, it is important for the nurse to assess for this complication of burn injuries. Psychosis, delirium, and dementia are not among the noted psychiatric and psychosocial complications of burns.

A nurse in a busy emergency department provides care for many patients who present with contusions, strains, or sprains. Treatment modalities that are common to all of these musculoskeletal injuries include which of the following? Select all that apply. A) Massage B) Applying ice C) Compression dressings D) Resting the affected extremity E) Corticosteroids F) Elevating the injured limb

Ans: B, C, D, F Feedback: Treatment of contusions, strains, and sprains consists of resting and elevating the affected part, applying cold, and using a compression bandage. Massage and corticosteroids are not used to treat these injuries.

A school nurse is assessing a student who was kicked in the shin during a soccer game. The area of the injury has become swollen and discolored. The triage nurse recognizes that the patient has likely sustained what? A) Sprain B) Strain C) Contusion D) Dislocation

Ans: C Feedback: A contusion is a soft-tissue injury that results in bleeding into soft tissues, creating a hematoma and ecchymosis. A sprain is an injury to ligaments caused by wrenching or twisting. A strain is a "muscle pull" from overuse, overstretching, or excessive stress. A dislocation is a condition in which the articular surfaces of the bones forming a joint are no longer in anatomic contact. Because the injury is not at the site of a joint, the patient has not experienced a sprain, strain, or dislocation.

A nurse is planning the care of a patient with osteomyelitis that resulted from a diabetic foot ulcer. The patient requires a transmetatarsal amputation. When planning the patient's postoperative care, which of the following nursing diagnoses should the nurse most likely include in the plan of care? A) Ineffective Thermoregulation B) Risk-Prone Health Behavior C) Disturbed Body Image D) Deficient Diversion Activity

Ans: C Feedback: Amputations present a serious threat to any patient's body image. None of the other listed diagnoses is specifically associated with amputation.

A patient has presented to the emergency department with an injury to the wrist. The patient is diagnosed with a third-degree strain. Why would the physician order an x-ray of the wrist? A) Nerve damage is associated with third-degree strains. B) Compartment syndrome is associated with third-degree strains. C) Avulsion fractures are associated with third-degree strains. D) Greenstick fractures are associated with third-degree strains.

Ans: C Feedback: An x-ray should be obtained to rule out bone injury, because an avulsion fracture (in which a bone fragment is pulled away from the bone by a tendon) may be associated with a third-degree strain. Nerve damage, compartment syndrome, and greenstick fractures are not associated with third-degree strains.

A nurse is caring for a patient who has suffered a hip fracture and who will require an extended hospital stay. The nurse should ensure that the patient does which of the following in order to prevent common complications associated with a hip fracture? A) Avoid requesting analgesia unless pain becomes unbearable. B) Use supplementary oxygen when transferring or mobilizing. C) Increase fluid intake and perform prescribed foot exercises. D) Remain on bed rest for 14 days or until instructed by the orthopedic surgeon.

Ans: C Feedback: Deep vein thrombosis (DVT) is among the most common complications related to a hip fracture. To prevent DVT, the nurse encourages intake of fluids and ankle and foot exercises. The patient should not be told to endure pain; a proactive approach to pain control should be adopted. While respiratory complications commonly include atelectasis and pneumonia, the use of deep-breathing exercises, changes in position at least every 2 hours, and the use of incentive spirometry help prevent the respiratory complications more than using supplementary oxygen. Bed rest may be indicated in the short term, but is not normally required for 14 days.

A nurse is performing a shift assessment on an elderly patient who is recovering after surgery for a hip fracture. The nurse notes that the patient is complaining of chest pain, has an increased heart rate, and increased respiratory rate. The nurse further notes that the patient is febrile and hypoxic, coughing, and producing large amounts of thick, white sputum. The nurse recognizes that this is a medical emergency and calls for assistance, recognizing that this patient is likely demonstrating symptoms of what complication? A) Avascular necrosis of bone B) Compartment syndrome C) Fat embolism syndrome D) Complex regional pain syndrome

Ans: C Feedback: Fat embolism syndrome occurs most frequently in young adults and elderly patients who experience fractures of the proximal femur (i.e., hip fracture). Presenting features of fat embolism syndrome include hypoxia, tachypnea, tachycardia, and pyrexia. The respiratory distress response includes tachypnea, dyspnea, wheezes, precordial chest pain, cough, large amounts of thick, white sputum, and tachycardia. Avascular necrosis (AVN) occurs when the bone loses its blood supply and dies. This does not cause coughing. Complex regional pain syndrome does not have cardiopulmonary involvement.

A patient has come to the orthopedic clinic for a follow-up appointment 6 weeks after fracturing his ankle. Diagnostic imaging reveals that bone union is not taking place. What factor may have contributed to this complication? A) Inadequate vitamin D intake B) Bleeding at the injury site C) Inadequate immobilization D) Venous thromboembolism (VTE)

Ans: C Feedback: Inadequate fracture immobilization can delay or prevent union. A short-term vitamin D deficiency would not likely prevent bone union. VTE is a serious complication but would not be a cause of nonunion. Similarly, bleeding would not likely delay union.

A 20 year-old is brought in by ambulance to the emergency department after being involved in a motorcycle accident. The patient has an open fracture of his tibia. The wound is highly contaminated and there is extensive soft-tissue damage. How would this patient's fracture likely be graded? A) Grade I B) Grade II C) Grade III D) Grade IV

Ans: C Feedback: Open fractures are graded according to the following criteria. Grade I is a clean wound less than 1 cm long. Grade II is a larger wound without extensive soft-tissue damage. Grade III is highly contaminated, has extensive soft-tissue damage, and is the most severe. There is no grade IV fracture.

Six weeks after an above-the-knee amputation (AKA), a patient returns to the outpatient office for a routine postoperative checkup. During the nurse's assessment, the patient reports symptoms of phantom pain. What should the nurse tell the patient to do to reduce the discomfort of the phantom pain? A) Apply intermittent hot compresses to the area of the amputation. B) Avoid activity until the pain subsides. C) Take opioid analgesics as ordered. D) Elevate the level of the amputation site.

Ans: C Feedback: Opioid analgesics may be effective in relieving phantom pain. Heat, immobility, and elevation are not noted to relieve this form of pain.

A young patient is being treated for a femoral fracture suffered in a snowboarding accident. The nurse's most recent assessment reveals that the patient is uncharacteristically confused. What diagnostic test should be performed on this patient? A) Electrolyte assessment B) Electrocardiogram C) Arterial blood gases D) Abdominal ultrasound

Ans: C Feedback: Subtle personality changes, restlessness, irritability, or confusion in a patient who has sustained a fracture are indications for immediate arterial blood gas studies due to the possibility of fat embolism syndrome. This assessment finding does not indicate an immediate need for electrolyte levels, an ECG, or abdominal ultrasound.

A patient is being treated for a fractured hip and the nurse is aware of the need to implement interventions to prevent muscle wasting and other complications of immobility. What intervention best addresses the patient's need for exercise? A) Performing gentle leg lifts with both legs B) Performing massage to stimulate circulation C) Encouraging frequent use of the overbed trapeze D) Encouraging the patient to log roll side to side once per hour

Ans: C Feedback: The patient is encouraged to exercise as much as possible by means of the overbed trapeze. This device helps strengthen the arms and shoulders in preparation for protected ambulation. Independent logrolling may result in injury due to the location of the fracture. Leg lifts would be contraindicated for the same reason. Massage by the nurse is not a substitute for exercise.

A patient is admitted to the orthopedic unit with a fractured femur after a motorcycle accident. The patient has been placed in traction until his femur can be rodded in surgery. For what early complications should the nurse monitor this patient? Select all that apply. A) Systemic infection B) Complex regional pain syndrome C) Deep vein thrombosis D) Compartment syndrome E) Fat embolism

Ans: C, D, E Feedback: Early complications include shock, fat embolism, compartment syndrome, and venous thromboemboli (deep vein thrombosis [DVT], pulmonary embolism [PE]). Infection and CRPS are later complications of fractures.

40. A nurse who provides care on a burn unit is preparing to apply a patients ordered topical antibiotic ointment. What action should the nurse perform when administering this medication? A) Apply the new ointment without disturbing the existing layer of ointment. B) Apply the ointment using a sterile tongue depressor. C) Apply a layer of ointment approximately 1/16 inch thick. D) Gently irrigate the wound bed after applying the antibiotic ointment.

Ans: C Feedback: After removing the old ointment from the wound bed, the nurse should apply a layer of ointment 1/16- inch thick using clean gloves. The wound would not be irrigated after application of new ointment.

14. A triage nurse in the emergency department (ED) receives a phone call from a frantic father who saw his 4-year-old child tip a pot of boiling water onto her chest. The father has called an ambulance. What would the nurse in the ED receiving the call instruct the father to do? A) Cover the burn with ice and secure with a towel. B) Apply butter to the area that is burned. C) Immerse the child in a cool bath. D) Avoid touching the burned area under any circumstances.

Ans: C Feedback: After the flames or heat source have been removed or extinguished, the burned area and adherent clothing are soaked with cool water briefly to cool the wound and halt the burning process. Cool water is the best first-aid measure. Ice and butter are contraindicated. Appropriate first aid necessitates touching the burn.

28. A patient experienced a 33% TBSA burn 72 hours ago. The nurse observes that the patients hourly urine output has been steadily increasing over the past 24 hours. How should the nurse best respond to this finding? A) Obtain an order to reduce the rate of the patients IV fluid infusion. B) Report the patients early signs of acute kidney injury (AKI). C) Recognize that the patient is experiencing an expected onset of diuresis. D) Administer sodium chloride as ordered to compensate for this fluid loss.

Ans: C Feedback: As capillaries regain integrity, 48 or more hours after the burn, fluid moves from the interstitial to the intravascular compartment and diuresis begins. This is an expected development and does not require a reduction in the IV infusion rate or the administration of NaCl. Diuresis is not suggestive of AKI.

24. A patient with severe burns is admitted to the intensive care unit to stabilize and begin fluid resuscitation before transport to the burn center. The nurse should monitor the patient closely for what signs of the onset of burn shock? A) Confusion B) High fever C) Decreased blood pressure D) Sudden agitation

Ans: C Feedback: As fluid loss continues and vascular volume decreases, cardiac output continues to decrease and the blood pressure drops, marking the onset of burn shock. Shock and the accompanying hemodynamic changes are not normally accompanied by confusion, fever, or agitation.

11. A patient with a partial-thickness burn injury had Biobrane applied 2 weeks ago. The nurse notices that the Biobrane is separating from the burn wound. What is the nurses most appropriate intervention? A) Reinforce the Biobrane dressing with another piece of Biobrane. B) Remove the Biobrane dressing and apply a new dressing. C) Trim away the separated Biobrane. D) Notify the physician for further emergency-related orders.

Ans: C Feedback: As the Biobrane gradually separates, it is trimmed, leaving a healed wound. When the Biobrane dressing adheres to the wound, the wound remains stable and the Biobrane can remain in place for 3 to 4 weeks. There is no need to reinforce the Biobrane nor to remove it and apply a new dressing. There is not likely any need to notify the physician for further orders.

31. A patient has experienced burns to his upper thighs and knees. Following the application of new wound dressings, the nurse should perform what nursing action? A) Instruct the patient to keep the wound site in a dependent position. B) Administer PRN analgesia as ordered. C) Assess the patients peripheral pulses distal to the dressing. D) Assist with passive range of motion exercises to set the new dressing.

Ans: C Feedback: Dressings can impede circulation if they are wrapped too tightly. The peripheral pulses must be checked frequently and burned extremities elevated. Dependent positioning does not need to be maintained. PRN analgesics should be administered prior to the dressing change. ROM exercises do not normally follow a dressing change.

15. A nurse is teaching a patient with a partial-thickness wound how to wear his elastic pressure garment. How would the nurse instruct the patient to wear this garment? A) 4 to 6 hours a day for 6 months B) During waking hours for 2 to 3 months after the injury C) Continuously D) At night while sleeping for a year after the injury

Ans: C Feedback: Elastic pressure garments are worn continuously (i.e., 23 hours a day).

2. The current phase of a patients treatment for a burn injury prioritizes wound care, nutritional support, and prevention of complications such as infection. Based on these care priorities, the patient is in what phase of burn care? A) Emergent B) Immediate resuscitative C) Acute D) Rehabilitation

Ans: C Feedback: The acute or intermediate phase of burn care follows the emergent/resuscitative phase and begins 48 to 72 hours after the burn injury. During this phase, attention is directed toward continued assessment and maintenance of respiratory and circulatory status, fluid and electrolyte balance, and gastrointestinal function. Infection prevention, burn wound care (i.e., wound cleaning, topical antibacterial therapy, wound dressing, dressing changes, wound debridement, and wound grafting), pain management, and nutritional support are priorities at this stage. Priorities during the emergent or immediate resuscitative phase include first aid, prevention of shock and respiratory distress, detection and treatment of concomitant injuries, and initial wound assessment and care. The priorities during the rehabilitation phase include prevention of scars and contractures, rehabilitation, functional and cosmetic reconstruction, and psychosocial counseling.

19. A nurse is developing a care plan for a patient with a partial-thickness burn, and determines that an appropriate goal is to maintain position of joints in alignment. What is the best rationale for this intervention? A) To prevent neuropathies B) To prevent wound breakdown C) To prevent contractures D) To prevent heterotopic ossification

Ans: C Feedback: To prevent the complication of contractures, the nurse will establish a goal to maintain position of joints in alignment. Gentle range of motion exercises and a consult to PT and OT for exercises and positioning recommendations are also appropriate interventions for the prevention of contractures. Joint alignment is not maintained specifically for preventing neuropathy, wound breakdown, or heterotopic ossification.

A bone biopsy has just been completed on a patient with suspected bone metastases. What assessment should the nurse prioritize in the immediate recovery period? A) Assessment for dehiscence at the biopsy site B) Assessment for pain C) Assessment for hematoma formation D) Assessment for infection

B

A nurse is preparing to discharge an emergency department patient who has been fitted with a sling to support her arm after a clavicle fracture. What should the nurse instruct the patient to do? A) Elevate the arm above the shoulder 3 to 4 times daily. B) Avoid moving the elbow, wrist, and fingers until bone remodeling is complete. C) Engage in active range of motion using the affected arm. D) Use the arm for light activities within the range of motion.

Ans: D Feedback: A patient with a clavicle fracture may use a sling to support the arm and relieve the pain. The patient may be permitted to use the arm for light activities within the range of comfort. The patient should not elevate the arm above the shoulder level until the ends of the bones have united, but the nurse should encourage the patient to exercise the elbow, wrist, and fingers.

Radiographs of a boy's upper arm show that the humerus appears to be fractured on one side and slightly bent on the other. This diagnostic result suggests what type of fracture? A) Impacted B) Compound C) Compression D) Greenstick

Ans: D Feedback: Greenstick fractures are an incomplete fracture that results in the bone being broken on one side, while the other side is bent. This is not characteristic of an impacted, compound, or compression fracture.

An emergency department patient is diagnosed with a hip dislocation. The patient's family is relieved that the patient has not suffered a hip fracture, but the nurse explains that this is still considered to be a medical emergency. What is the rationale for the nurse's statement? A) The longer the joint is displaced, the more difficult it is to get it back in place. B) The patient's pain will increase until the joint is realigned. C) Dislocation can become permanent if the process of bone remodeling begins. D) Avascular necrosis may develop at the site of the dislocation if it is not promptly resolved.

Ans: D Feedback: If a dislocation or subluxation is not reduced immediately, avascular necrosis (AVN) may develop. Bone remodeling does not take place because a fracture has not occurred. Realignment does not become more difficult with time and pain would subside with time, not become worse.

A patient is brought to the emergency department by ambulance after stepping in a hole and falling. While assessing him the nurse notes that his right leg is shorter than his left leg; his right hip is noticeably deformed and he is in acute pain. Imaging does not reveal a fracture. Which of the following is the most plausible explanation for this patient's signs and symptoms? A) Subluxated right hip B) Right hip contusion C) Hip strain D) Traumatic hip dislocation

Ans: D Feedback: Signs and symptoms of a traumatic dislocation include acute pain, change in positioning of the joint, shortening of the extremity, deformity, and decreased mobility. A subluxation would cause moderate deformity, or possibly no deformity. A contusion or strain would not cause obvious deformities.

A patient who has had an amputation is being cared for by a multidisciplinary rehabilitation team. What is the primary goal of this multidisciplinary team? A) Maximize the efficiency of care B) Ensure that the patient's health care is holistic C) Facilitate the patient's adjustment to a new body image D) Promote the patient's highest possible level of function

Ans: D Feedback: The multidisciplinary rehabilitation team helps the patient achieve the highest possible level of function and participation in life activities. The team is not primarily motivated by efficiency, the need for holistic care, or the need to foster the patient's body image, despite the fact that each of these are valid goals.

A nurse is preparing to discharge a patient from the emergency department after receiving treatment for an ankle sprain. While providing discharge education, the nurse should encourage which of the following? A) Apply heat for the first 24 to 48 hours after the injury. B) Maintain the ankle in a dependent position. C) Exercise hourly by performing rotation exercises of the ankle. D) Keep an elastic compression bandage on the ankle.

Ans: D Feedback: Treatment of a sprain consists of resting and elevating the affected part, applying cold, and using a compression bandage. After the acute inflammatory stage (usually 24 to 48 hours after injury), heat may be applied intermittently. Rotation exercises would likely be painful.

1. A patient is brought to the emergency department from the site of a chemical fire, where he suffered a burn that involves the epidermis, dermis, and the muscle and bone of the right arm. On inspection, the skin appears charred. Based on these assessment findings, what is the depth of the burn on the patients arm? A) Superficial partial-thickness B) Deep partial-thickness C) Full partial-thickness D) Full-thickness

Ans: D Feedback: A full-thickness burn involves total destruction of the epidermis and dermis and, in some cases, underlying tissue as well. Wound color ranges widely from white to red, brown, or black. The burned area is painless because the nerve fibers are destroyed. The wound can appear leathery; hair follicles and sweat glands are destroyed. Edema may also be present. Superficial partial-thickness burns involve the epidermis and possibly a portion of the dermis; the patient will experience pain that is soothed by cooling. Deep partial-thickness burns involve the epidermis, upper dermis, and portion of the deeper dermis; the patient will complain of pain and sensitivity to cold air. Full partial thickness is not a depth of burn.

37. The nurse caring for a patient who is recovering from full-thickness burns is aware of the patients risk for contracture and hypertrophic scarring. How can the nurse best mitigate this risk? A) Apply skin emollients as ordered after granulation has occurred. B) Keep injured areas immobilized whenever possible to promote healing. C) Administer oral or IV corticosteroids as ordered. D) Encourage physical activity and range of motion exercises.

Ans: D Feedback: Exercise and the promotion of mobility can reduce the risk of contracture and hypertrophic scarring. Skin emollients are not normally used in the treatment of burns, and these do not prevent scarring. Steroids are not used to reduce scarring, as they also slow the healing process.

26. A patients burns are estimated at 36% of total body surface area; fluid resuscitation has been ordered in the emergency department. After establishing intravenous access, the nurse should anticipate the administration of what fluid? A) 0.45% NaCl with 20 mEq/L KCl B) 0.45% NaCl with 40 mEq/L KCl C) Normal saline D) Lactated Ringers

Ans: D Feedback: Fluid resuscitation with lactated Ringers (LR) should be initiated using the American Burn Associations (ABA) fluid resuscitation formulas. LR is the crystalloid of choice because its composition and osmolality most closely resemble plasma and because use of normal saline is associated with hyperchloremic acidosis. Potassium chloride solutions would exacerbate the hyperkalemia that occurs following burn injuries.

5. An occupational health nurse is called to the floor of a factory where a worker has sustained a flash burn to the right arm. The nurse arrives and the flames have been extinguished. The next step is to cool the burn. How should the nurse cool the burn? A) Apply ice to the site of the burn for 5 to 10 minutes. B) Wrap the patients affected extremity in ice until help arrives. C) Apply an oil-based substance or butter to the burned area until help arrives. D) Wrap cool towels around the affected extremity intermittently.

Ans: D Feedback: Once the burn has been sustained, the application of cool water is the best first-aid measure. Soaking the burn area intermittently in cool water or applying cool towels gives immediate and striking relief from pain, and limits local tissue edema and damage. However, never apply ice directly to the burn, never wrap the person in ice, and never use cold soaks or dressings for longer than several minutes; such procedures may worsen the tissue damage and lead to hypothermia in people with large burns. Butter is contraindicated.

13. A nurse is caring for a patient who has sustained a deep partial-thickness burn injury. In prioritizing the nursing diagnoses for the plan of care, the nurse will give the highest priority to what nursing diagnosis? A) Activity Intolerance B) Anxiety C) Ineffective Coping D) Acute Pain

Ans: D Feedback: Pain is inevitable during recovery from any burn injury. Pain in the burn patient has been described as one of the most severe causes of acute pain. Management of the often-severe pain is one of the most difficult challenges facing the burn team. While the other nursing diagnoses listed are valid, the presence of pain may contribute to these diagnoses. Management of the patients pain is the priority, as it may have a direct correlation to the other listed nursing diagnoses.

32. A nurse is caring for a patient with burns who is in the later stages of the acute phase of recovery. The plan of nursing care should include which of the following nursing actions? A) Maintenance of bed rest to aid healing B) Choosing appropriate splints and functional devices C) Administration of beta adrenergic blockers D) Prevention of venous thromboembolism

Ans: D Feedback: Prevention of deep vein thrombosis (DVT) is an important factor in care. Early mobilization of the patient is important. The nurse monitors the splints and functional devices, but these are selected by occupational and physical therapists. The hemodynamic changes accompanying burns do not normally require the use of beta blockers.

30. A nurse is performing a home visit to a patient who is recovering following a long course of inpatient treatment for burn injuries. When performing this home visit, the nurse should do which of the following? A) Assess the patient for signs of electrolyte imbalances. B) Administer fluids as ordered. C) Assess the risk for injury recurrence. D) Assess the patients psychosocial state.

Ans: D Feedback: Recovery from burns can be psychologically challenging; the nurses assessments must address this reality. Fluid and electrolyte imbalances are infrequent during the rehabilitation phase of recovery. Burns are not typically a health problem that tends to recur; the experience of being burned tends to foster vigilance.

6. An emergency department nurse has just admitted a patient with a burn. What characteristic of the burn will primarily determine whether the patient experiences a systemic response to this injury? A) The length of time since the burn B) The location of burned skin surfaces C) The source of the burn D) The total body surface area (TBSA) affected by the burn

Ans: D Feedback: Systemic effects are a result of several variables. However, TBSA and wound severity are considered the major factors that affect the presence or absence of systemic effects.

16. A patient is brought to the ED by paramedics, who report that the patient has partial-thickness burns on the chest and legs. The patient has also suffered smoke inhalation. What is the priority in the care of a patient who has been burned and suffered smoke inhalation? A) Pain B) Fluid balance C) Anxiety and fear D) Airway management

Ans: D Feedback: Systemic threats from a burn are the greatest threat to life. The ABCs of all trauma care apply during the early postburn period. While all options should be addressed, pain, fluid balance, and anxiety and fear do not take precedence over airway management.

38. While performing a patients ordered wound care for the treatment of a burn, the patient has made a series of sarcastic remarks to the nurse and criticized her technique. How should the nurse best interpret this patients behavior? A) The patient may be experiencing an adverse drug reaction that is affecting his cognition and behavior. B) The patient may be experiencing neurologic or psychiatric complications of his injuries. C) The patient may be experiencing inconsistencies in the care that he is being provided. D) The patient may be experiencing anger about his circumstances that he is deflecting toward the nurse.

Ans: D Feedback: The patient may experience feelings of anger. The anger may be directed outward toward those who escaped unharmed or toward those who are now providing care. While drug reactions, complications, and frustrating inconsistencies in care cannot be automatically ruled out, it is not uncommon for anger to be directed at caregivers.

17. A patient arrives in the emergency department after being burned in a house fire. The patients burns cover the face and the left forearm. What extent of burns does the patient most likely have? A) 13% B) 25% C) 9% D) 18%

Ans: D Feedback: When estimating the percentage of body area or burn surface area that has been burned, the Rule of Nines is used: the face is 9%, and the forearm is 9% for a total of 18% in this patient.

The nurse is planning patient teaching for a patient who is scheduled for an open hemicolectomy. The nurse intends to address the topics of incision splinting and leg exercises during this teaching session. when is the best to provide teaching? Upon the patients admission to the postanesthesia care unit (PACU) When the patient returns from the PACU During the intraoperative period As soon as possible before the surgical procedure

As soon as possible before the surgical procedure

A dermatologist has asked the nurse to assist with examination of a patients skin using a Woods light. This test will allow the physician to assess for which of the following? A) The presence of minute regions of keloid scarring B) Unusual patterns of pigmentation on the patients skin C) Vascular lesions that are not visible to the naked eye D) The presence of parasites on the epidermis

B

A gerontologic nurse is teaching a group of nursing students about integumentary changes that occur in older adults. How should these students best integrate these changes into care planning? A) By avoiding the use of moisturizing lotions on older adults skin B) By protecting older adults against shearing injuries C) By avoiding the use of ice packs to treat muscle pain D) By protecting older adults against excessive sweat accumulation

B

A nurse is assessing a patient who is experiencing peripheral neurovascular dysfunction. What assessment findings are most consistent with this diagnosis? A) Hot skin with a capillary refill of 1 to 2 seconds B) Absence of feeling, capillary refill of 4 to 5 seconds, and cool skin C) Pain, diaphoresis, and erythema D) Jaundiced skin, weakness, and capillary refill of 3 seconds

B

A nurse is caring for an older adult who has been diagnosed with geriatric failure to thrive. This patients prolonged immobility creates a risk for what complication? A) Muscle clonus B) Muscle atrophy C) Rheumatoid arthritis D) Muscle fasciculations

B

A nurse is doing a shift assessment on a group of patients after first taking report. An elderly patient is having her second dose of IV antibiotics for a diagnosis of pneumonia. The nurse notices a new rash on the patients chest. The nurse should ask what priority question regarding the presence of a reddened rash? A) Is the rash worse at a particular time or season? B) Are you allergic to any foods or medication? C) Are you having any loss of sensation in that area? D) Is your rash painful?

B

A nurse is explaining the importance of sunlight on the skin to a woman with decreased mobility who rarely leaves her house. The nurse would emphasize that ultraviolet light helps to synthesize what vitamin? A) E B) D C) A D) C

B

A nurse is performing a nursing assessment of a patient suspected of having a musculoskeletal disorder. What is the primary focus of the nursing assessment with a patient who has a musculoskeletal disorder? A) Range of motion B) Activities of daily living C) Gait D) Strength

B

A nurse practitioner working in a dermatology clinic finds an open lesion on a patient who is being assessed. What should the nurse do next? A) Obtain a swab for culture. B) Assess the characteristics of the lesion. C) Obtain a swab for pH testing. D) Apply a test dose of broad-spectrum topical antibiotic.

B

A patient is diagnosed with atrial fibrillation and the physician orders Coumadin (warfarin). For what skin lesion should the nurse monitor this patient? A) Ulcer B) Ecchymosis C) Scar D) Erosion

B

A patient is receiving ongoing nursing care for the treatment of Parkinsons disease. When assessing this patients gait, what finding is most closely associated with this health problem? A) Spastic hemiparesis gait B) Shuffling gait C) Rapid gait D) Steppage gait

B

A patient is scheduled for a bone scan to rule out osteosarcoma of the pelvic bones. What would be most important for the nurse to assess before the patients scan? A) That the patient completed the bowel cleansing regimen B) That the patient emptied the bladder C) That the patient is not allergic to penicillins D) That the patient has fasted for at least 8 hours

B

A patient is suspected of developing an allergy to an environmental substance and has been given a patch test. During the test, the patient develops fine blisters, papules, and severe itching. The nurse knows that this is indicative of what strength reaction? A) Weak positive B) Moderately positive C) Strong positive D) Severely positive

B

A patient is undergoing diagnostic testing for suspected Pagets disease. What assessment finding is most consistent with this diagnosis? A) Altered serum magnesium levels B) Altered serum calcium levels C) Altered serum potassium levels D) Altered serum sodium levels

B

A wound care nurse is reviewing skin anatomy with a group of medical nurses. Which area of the skin would the nurse identify as providing a cushion between the skin layers, muscles, and bones? A) Dermis B) Subcutaneous tissue C) Epidermis D) Stratum corneum

B

Assessment of a patients leg reveals the presence of a 1.5-cm circular region of necrotic tissue that is deeper than the epidermis. The nurse should document the presence of what type of skin lesion? A) Keloid B) Ulcer C) Fissure D) Erosion

B

When assessing a patients peripheral nerve function, the nurse uses an instrument to prick the fat pad at the top of the patients small finger. This action will assess which of the following nerves? A) Radial B) Ulnar C) Median D) Tibial

B

17. The nurse is planning the care of a patient with hyperthyroidism. What should the nurse specify in the patients meal plan? A) A clear liquid diet, high in nutrients B) Small, frequent meals, high in protein and calories C) Three large, bland meals a day D) A diet high in fiber and plant-sourced fat

B Feedback: A patient with hyperthyroidism has an increased appetite. The patient should be counseled to consume several small, well-balanced meals. High-calorie, high-protein foods are encouraged. A clear liquid diet would not satisfy the patients caloric or hunger needs. A diet rich in fiber and fat should be avoided because these foods may lead to GI upset or increase peristalsis.

5. The nurse is caring for a patient with Addisons disease who is scheduled for discharge. When teaching the patient about hormone replacement therapy, the nurse should address what topic? A) The possibility of precipitous weight gain B) The need for lifelong steroid replacement C) The need to match the daily steroid dose to immediate symptoms D) The importance of monitoring liver function

B Feedback: Because of the need for lifelong replacement of adrenal cortex hormones to prevent addisonian crises, the patient and family members receive explicit education about the rationale for replacement therapy and proper dosage. Doses are not adjusted on a short-term basis. Weight gain and hepatotoxicity are not common adverse effects.

15. While assisting with the surgical removal of an adrenal tumor, the OR nurse is aware that the patients vital signs may change upon manipulation of the tumor. What vital sign changes would the nurse expect to see? A) Hyperthermia and tachypnea B) Hypertension and heart rate changes C) Hypotension and hypothermia D) Hyperthermia and bradycardia

B Feedback: Manipulation of the tumor during surgical excision may cause release of stored epinephrine and norepinephrine, with marked increases in BP and changes in heart rate. The use of sodium nitroprusside and alpha-adrenergic blocking agents may be required during and after surgery. While other vital sign changes may occur related to surgical complications, the most common changes are related to hypertension and changes in the heart rate.

39. A patient who has been taking corticosteroids for several months has been experiencing muscle wasting. The patient has asked the nurse for suggestions to address this adverse effect. What should the nurse recommend? A) Activity limitation to conserve energy B) Consumption of a high-protein diet C) Use of OTC vitamin D and calcium supplements D) Passive range-of-motion exercises

B Feedback: Muscle wasting can be partly addressed through increased protein intake. Passive ROM exercises maintain flexibility, but do not build muscle mass. Vitamin D and calcium supplements do not decrease muscle wasting. Activity limitation would exacerbate the problem.

22. The physician has ordered a fluid deprivation test for a patient suspected of having diabetes insipidus. During the test, the nurse should prioritize what assessments? A) Temperature and oxygen saturation B) Heart rate and BP C) Breath sounds and bowel sounds D) Color, warmth, movement, and sensation of extremities

B Feedback: The fluid deprivation test is carried out by withholding fluids for 8 to 12 hours or until 3% to 5% of the body weight is lost. The patients condition needs to be monitored frequently during the test, and the test is terminated if tachycardia, excessive weight loss, or hypotension develops. Consequently, BP and heart rate monitoring are priorities over the other listed assessments.

36. What should the nurse teach a patient on corticosteroid therapy in order to reduce the patients risk of adrenal insufficiency? A) Take the medication late in the day to mimic the bodys natural rhythms. B) Always have enough medication on hand to avoid running out. C) Skip up to 2 doses in cases of illness involving nausea. D) Take up to 1 extra dose per day during times of stress.

B Feedback: The patient and family should be informed that acute adrenal insufficiency and underlying symptoms will recur if corticosteroid therapy is stopped abruptly without medical supervision. The patient should be instructed to have an adequate supply of the corticosteroid medication always available to avoid running out. Doses should not be skipped or added without explicit instructions to do so. Corticosteroids should normally be taken in the morning to mimic natural rhythms.

18. A patient with a diagnosis of syndrome of inappropriate antidiuretic hormone secretion (SIADH) is being cared for on the critical care unit. The priority nursing diagnosis for a patient with this condition is what? A) Risk for peripheral neurovascular dysfunction B) Excess fluid volume C) Hypothermia D) Ineffective airway clearance

B Feedback: The priority nursing diagnosis for a patient with SIADH is excess fluid volume, as the patient retains fluids and develops a sodium deficiency. Restricting fluid intake is a typical intervention for managing this syndrome. Temperature imbalances are not associated with SIADH. The patient is not at risk for neurovascular dysfunction or a compromised airway.

26. The nurses assessment of a patient with thyroidectomy suggests tetany and a review of the most recent blood work corroborate this finding. The nurse should prepare to administer what intervention? A) Oral calcium chloride and vitamin D B) IV calcium gluconate C) STAT levothyroxine D) Administration of parathyroid hormone (PTH)

B Feedback: When hypocalcemia and tetany occur after a thyroidectomy, the immediate treatment is administration of IV calcium gluconate. This has a much faster therapeutic effect than PO calcium or vitamin D supplements. PTH and levothyroxine are not used to treat this complication.

A medical nurse is caring for a patient with type 1 diabetes. The patients medication administration record includes the administration of regular insulin three times daily. Knowing that the patients lunch tray will arrive at 11:45, when should the nurse administer the patients insulin? A) 10:45 B) 11:15 C) 11:45 D) 11:50

B) 11:15

A patient has just been diagnosed with type 2 diabetes. The physician has prescribed an oral antidiabetic agent that will inhibit the production of glucose by the liver and thereby aid in the control of blood glucose. What type of oral antidiabetic agent did the physician prescribe for this patient? A) A sulfonylurea B) A biguanide C) A thiazolidinedione D) An alpha glucosidase inhibitor

B) A biguanide

A patient has been admitted to the critical care unit with a diagnosis of thyroid storm. What interventions should the nurse include in this patient's immediate care? Select all that apply. A) Administering diuretics to prevent fluid overload B) Administering beta blockers to reduce heart rate C) Administering insulin to reduce blood glucose levels D) Applying interventions to reduce the patient's temperature E) Administering corticosteroids

B) Administering beta blockers to reduce heart rate D) Applying interventions to reduce the patient's temperature

A diabetes nurse is assessing a patients knowledge of self-care skills. What would be the most appropriate way for the educator to assess the patients knowledge of nutritional therapy in diabetes? A) Ask the patient to describe an optimally healthy meal. B) Ask the patient to keep a food diary and review it with the nurse. C) Ask the patients family what he typically eats. D) Ask the patient to describe a typical days food intake.

B) Ask the patient to keep a food diary and review it with the nurse.

An elderly patient comes to the clinic with her daughter. The patient is a diabetic and is concerned about foot care. The nurse goes over foot care with the patient and her daughter as the nurse realizes that foot care is extremely important. Why would the nurse feel that foot care is so important to this patient? A) An elderly patient with foot ulcers experiences severe foot pain due to the diabetic polyneuropathy. B) Avoiding foot ulcers may mean the difference between institutionalization and continued independent living. C) Hypoglycemia is linked with a risk for falls; this risk is elevated in older adults with diabetes. D) Oral antihyperglycemics have the possible adverse effect of decreased circulation to the lower extremities.

B) Avoiding foot ulcers may mean the difference between institutionalization and continued independent living.

A patient who has been taking corticosteroids for several months has been experiencing muscle wasting. The patient has asked the nurse for suggestions to address this adverse effect. What should the nurse recommend? A) Activity limitation to conserve energy B) Consumption of a high-protein diet C) Use of OTC vitamin D and calcium D) Passive range-of-motion exercises

B) Consumption of a high-protein diet

A patient with a diagnosis of syndrome of inappropriate antidiuretic hormone secretion (SIADH) is being cared for on the critical care unit. The priority nursing diagnosis for a patient with this condition is what? A) Risk for peripheral neurovascular dysfunction B) Excess fluid volume C) Hypothermia D) Ineffective airway clearance

B) Excess fluid volume

The physician has ordered a fluid deprivation test for a patient suspected of having diabetes insipidus. During the test, the nurse should prioritize what assessments? A) Temperature and oxygen saturation B) Heart rate and BP C) Breath sounds and bowel sounds D) Color, warmth, movement, and sensation of extremities

B) Heart rate and BP

While assisting with the surgical removal of an adrenal tumor, the OR nurse is aware that the patient's vital signs may change upon manipulation of the tumor. What vital sign changes would the nurse expect to see? A) Hyperthermia and tachypnea B) Hypertension and heart rate changes C) Hypotension and hypothermia D) Hyperthermia and bradycardia

B) Hypertension and heart rate changes

The nurse's assessment of a patient with thyroidectomy suggests tetany and a review of the most recent blood work corroborate this finding. The nurse should prepare to administer what intervention? A) Oral calcium chloride and vitamin D B) IV calcium gluconate C) STAT levothyroxine D) Administration of parathyroid hormone(PTH)

B) IV calcium gluconate

An occupational health nurse is screening a group of workers for diabetes. What statement should the nurse interpret as suggestive of diabetes? A) Ive always been a fan of sweet foods, but lately Im turned off by them. B) Lately, I drink and drink and cant seem to quench my thirst. C) No matter how much sleep I get, it seems to take me hours to wake up. D) When I went to the washroom the last few days, my urine smelled odd

B) Lately, I drink and drink and cant seem to quench my thirst.

A patient with a history of type 1 diabetes has just been admitted to the critical care unit (CCU) for diabetic ketoacidosis. The CCU nurse should prioritize what assessment during the patients initial phase of treatment? A) Monitoring the patient for dysrhythmias B) Maintaining and monitoring the patients fluid balance C) Assessing the patients level of consciousness D) Assessing the patient for signs and symptoms of venous thromboembolism

B) Maintaining and monitoring the patients fluid balance

A nurse is caring for a patient with type 1 diabetes who is being discharged home tomorrow. What is the best way to assess the patients ability to prepare and self-administer insulin? A) Ask the patient to describe the process in detail. B) Observe the patient drawing up and administering the insulin. C) Provide a health education session reviewing the main points of insulin delivery. D) Review the patients first hemoglobin A1C result after discharge.

B) Observe the patient drawing up and administering the insulin.

The nurse is caring for a patient at risk for an addisonian cirisis. For what associated signs and symptoms should the nurse monitor the patient? Select all that apply. A) Epitaxis B) Pallor C) Rapid respiratory rate D) Bounding pulse E) Hypotension

B) Pallor C) Rapid respiratory rate E) Hypotension

The nurse is planning the care of a patient with hyperthyroidism. What should the nurse specify in the patient's meal plan? A) A clear thin liquid diet, high in nutrients B) Small frequent meals, high in protein and calories C) Three large, bland meals a day D) A diet high in fiber and plant-sourced fat

B) Small frequent meals, high in protein and calories

A patient with type 2 diabetes achieves adequate glycemic control through diet and exercise. Upon being admitted to the hospital for a cholecystectomy, however, the patient has required insulin injections on two occasions. The nurse would identify what likely cause for this short-term change in treatment? A) Alterations in bile metabolism and release have likely caused hyperglycemia. B) Stress has likely caused an increase in the patients blood sugar levels. C) The patient has likely overestimated her ability to control her diabetes using nonpharmacologic measures. D) The patients volatile fluid balance surrounding surgery has likely caused unstable blood sugars.

B) Stress has likely caused an increase in the patients blood sugar levels.

The nurse is discussing macrovascular complications of diabetes with a patient. The nurse would address what topic during this dialogue? A) The need for frequent eye examinations for patients with diabetes B) The fact that patients with diabetes have an elevated risk of myocardial infarction C) The relationship between kidney function and blood glucose levels D) The need to monitor urine for the presence of albumin

B) The fact that patients with diabetes have an elevated risk of myocardial infarction

The nurse is caring for a patient with Addison's disease who is scheduled for discharge. When teaching the patient about hormone replacement therapy, the nurse should address what topic? A) The possibility of precipitous weight gain B) The need for lifelong steroid replacement C) The need to match the daily steroid dose to immediate symptoms D) The importance of monitoring liver function

B) The need for lifelong steroid replacement

Urinary anti-infectives are used only to treat urinary tract infections (UTIs). What causes urinary anti-infectives to be so effective in treating UTIs? A) They sterilize feces B) They act specifically within the urinary tract C) They reach high plasma levels in short period of time D) They are excreted through the liver

B) They act specifically with the urinary tract

A diabetic patient calls the clinic complaining of having a flu bug. The nurse tells him to take his regular dose of insulin. What else should the nurse tell the patient? A) Make sure to stick to your normal diet. B) Try to eat small amounts of carbs, if possible. C) Ensure that you check your blood glucose every hour. D) For now, check your urine for ketones every 8 hours.

B) Try to eat small amounts of carbs, if possible.

A patient is brought to the emergency department by the paramedics. The patient is a type 2 diabetic and is experiencing HHS. The nurse should identify what components of HHS? Select all that apply. A) Leukocytosis B) Glycosuria C) Dehydration D) Hypernatremia E) Hyperglycemia

B, C, D, E

37. The nurse is caring for a patient at risk for an addisonian crisis. For what associated signs and symptoms should the nurse monitor the patient? Select all that apply. A) Epistaxis B) Pallor C) Rapid respiratory rate D) Bounding pulse E) Hypotension

B, C, E Feedback: The patient at risk is monitored for signs and symptoms indicative of addisonian crisis, which can include shock; hypotension; rapid, weak pulse; rapid respiratory rate; pallor; and extreme weakness. Epistaxis and a bounding pulse are not symptoms or signs of an addisonian crisis.

25. A patient has been admitted to the critical care unit with a diagnosis of thyroid storm. What interventions should the nurse include in this patients immediate care? Select all that apply. A) Administering diuretics to prevent fluid overload B) Administering beta blockers to reduce heart rate C) Administering insulin to reduce blood glucose levels D) Applying interventions to reduce the patients temperature E) Administering corticosteroids

B, D Feedback: Thyroid storm necessitates interventions to reduce heart rate and temperature. Diuretics, insulin, and steroids are not indicated to address the manifestations of this health problem.

A nurse is seeing a 16-year-old male patient who has come to the dermatology clinic for treatment of acne. The nurse practitioner would know that the treatment may consist of which of the following medications A) Acyclovir (Zovirax) B) Benzoyl peroxide and erythromycin (Benzamycin) C) Diphenhydramine (Benadryl) D) Triamcinolone (Kenalog)

Benzoyl peroxide and erythromycin Benzamycin Benzamycin gel is among the topical treatments available for acne.

The nurse is performing a comprehensive assessment of a patients skin surfaces and intends to assess moisture, temperature, and texture. The nurse should perform this component of assessment in what way? A) By examining the patient under a Woods light B) By inspecting the patients skin in direct sunlight C) By palpating the patients skin D) By performing percussion of major skin surfaces

C

The nurse is performing an initial assessment of a patient who has a raised, pruritic rash. The patient denies taking any prescription medication and denies any allergies. What would be an appropriate question to ask this patient at this time? A) Is anyone in your family allergic to anything? B) How long have you had this abrasion? C) Do you take any over-the-counter drugs or herbal preparations? D) What do you do for a living?

C

The nurses comprehensive assessment of an older adult involves the assessment of the patients gait. How should the nurse best perform this assessment? A) Instruct the patient to walk heel-to-toe for 15 to 20 steps. B) Instruct the patient to walk in a straight line while not looking at the floor. C) Instruct the patient to walk away from the nurse for a short distance and then toward the nurse. D) Instruct the patient to balance on one foot for as long as possible and then walk in a circle around the room.

C

The results of a nurses musculoskeletal examination show an increase in the lumbar curvature of the spine. The nurse should recognize the presence of what health problem? A) Osteoporosis B) Kyphosis C) Lordosis D) Scoliosis

C

When planning the skin care of a patient with decreased mobility, the nurse is aware of the varying thickness of the epidermis. At what location is the epidermal layer thickest? A) The scalp B) The elbows C) The palms of the hands D) The knees

C

While assessing a 25-year-old female, the nurse notes that the patient has hair on her lower abdomen. Earlier in the health interview, the patient stated that her menses are irregular. The nurse should suspect what type of health problem? A) A metabolic disorder B) A malignancy C) A hormonal imbalance D) An infectious process

C

20. The nurse is assessing a patient diagnosed with Graves disease. What physical characteristics of Graves disease would the nurse expect to find? A) Hair loss B) Moon face C) Bulging eyes D) Fatigue

C Feedback: Clinical manifestations of the endocrine disorder Graves disease include exophthalmos (bulging eyes) and fine tremor in the hands. Graves disease is not associated with hair loss, a moon face, or fatigue.

29. A 30 year-old female patient has been diagnosed with Cushing syndrome. What psychosocial nursing diagnosis should the nurse most likely prioritize when planning the patients care? A) Decisional conflict related to treatment options B) Spiritual distress related to changes in cognitive function C) Disturbed body image related to changes in physical appearance D) Powerlessness related to disease progression

C Feedback: Cushing syndrome causes characteristic physical changes that are likely to result in disturbed body image. Decisional conflict and powerless may exist, but disturbed body image is more likely to be present. Cognitive changes take place in patients with Cushing syndrome, but these may or may not cause spiritual distress.

9. The nurse caring for a patient with Cushing syndrome is describing the dexamethasone suppression test scheduled for tomorrow. What does the nurse explain that this test will involve? A) Administration of dexamethasone orally, followed by a plasma cortisol level every hour for 3 hours B) Administration of dexamethasone IV, followed by an x-ray of the adrenal glands C) Administration of dexamethasone orally at 11 PM, and a plasma cortisol level at 8 AM the next morning D) Administration of dexamethasone intravenously, followed by a plasma cortisol level 3 hours after the drug is administered

C Feedback: Dexamethasone (1 mg) is administered orally at 11 PM, and a plasma cortisol level is obtained at 8 AM the next morning. This test can be performed on an outpatient basis and is the most widely used and sensitive screening test for diagnosis of pituitary and adrenal causes of Cushing syndrome.

16. A patient has returned to the floor after having a thyroidectomy for thyroid cancer. The nurse knows that sometimes during thyroid surgery the parathyroid glands can be injured or removed. What laboratory finding may be an early indication of parathyroid gland injury or removal? A) Hyponatremia B) Hypophosphatemia C) Hypocalcemia D) Hypokalemia

C Feedback: Injury or removal of the parathyroid glands may produce a disturbance in calcium metabolism and result in a decline of calcium levels (hypocalcemia). As the blood calcium levels fall, hyperirritability of the nerves occurs, with spasms of the hands and feet and muscle twitching. This group of symptoms is known as tetany and must be reported to the physician immediately, because laryngospasm may occur and obstruct the airway. Hypophosphatemia, hyponatremia, and hypokalemia are not expected responses to parathyroid injury or removal. In fact, parathyroid removal or injury that results in hypocalcemia may lead to hyperphosphatemia.

4. The nurse is caring for a patient with a diagnosis of Addisons disease. What sign or symptom is most closely associated with this health problem? A) Truncal obesity B) Hypertension C) Muscle weakness D) Moon face

C Feedback: Patients with Addisons disease demonstrate muscular weakness, anorexia, gastrointestinal symptoms, fatigue, emaciation, dark pigmentation of the skin, and hypotension. Patients with Cushing syndrome demonstrate truncal obesity, moon face, acne, abdominal striae, and hypertension.

8. A nurse caring for a patient with diabetes insipidus is reviewing laboratory results. What is an expected urinalysis finding? A) Glucose in the urine B) Albumin in the urine C) Highly dilute urine D) Leukocytes in the urine

C Feedback: Patients with diabetes insipidus produce an enormous daily output of very dilute, water-like urine with a specific gravity of 1.001 to 1.005. The urine contains no abnormal substances such as glucose or albumin. Leukocytes in the urine are not related to the condition of diabetes insipidus, but would indicate a urinary tract infection, if present in the urine.

6. The nurse is teaching a patient that the body needs iodine for the thyroid to function. What food would be the best source of iodine for the body? A) Eggs B) Shellfish C) Table salt D) Red meat

C Feedback: The major use of iodine in the body is by the thyroid. Iodized table salt is the best source of iodine.

7. A patient is prescribed corticosteroid therapy. What would be priority information for the nurse to give the patient who is prescribed long-term corticosteroid therapy? A) The patients diet should be low protein with ample fat. B) The patient may experience short-term changes in cognition. C) The patient is at an increased risk for developing infection. D) The patient is at a decreased risk for development of thrombophlebitis and thromboembolism.

C Feedback: The patient is at increased risk of infection and masking of signs of infection. The cardiovascular effects of corticosteroid therapy may result in development of thrombophlebitis or thromboembolism. Diet should be high in protein with limited fat. Changes in appearance usually disappear when therapy is no longer necessary. Cognitive changes are not common adverse effects.

2. A patient has been admitted to the post-surgical unit following a thyroidectomy. To promote comfort and safety, how should the nurse best position the patient? A) Side-lying (lateral) with one pillow under the head B) Head of the bed elevated 30 degrees and no pillows placed under the head C) Semi-Fowlers with the head supported on two pillows D) Supine, with a small roll supporting the neck

C Feedback: When moving and turning the patient, the nurse carefully supports the patients head and avoids tension on the sutures. The most comfortable position is the semi-Fowlers position, with the head elevated and supported by pillows.

Which of the following patients with type 1 diabetes is most likely to experience adequate glucose control? A) A patient who skips breakfast when his glucose reading is greater than 220 mg/dL B) A patient who never deviates from her prescribed dose of insulin C) A patient who adheres closely to a meal plan and meal schedule D) A patient who eliminates carbohydrates from his daily intake

C) A patient who adheres closely to a meal plan and meal schedule

The nurse caring for a patient with Cushing's syndrome is describing the dexamethasone suppression test scheduled for tomorrow. What does the nurse explain that this test will involve? A) Administration of dexamethasone orally, followed by a plasma cortisol level every hour for 3 hours B) Administration of dexamethasone IV, followed by an x-ray of the adrenal glands C) Administration of dexamethasone orally at 11PM, and a plasma cortisol level at 8AM the next morning D) Administration of dexamethasone intravenously, followed by a plasma cortisol level 3 hours after the drug is administered

C) Administration of dexamethasone orally at 11PM, and a plasma cortisol level at 8AM the next morning

A patient with type 1 diabetes mellitus is seeing the nurse to review foot care. What would be a priority instruction for the nurse to give the patient? A) Examine feet weekly for redness, blisters, and abrasions. B) Avoid the use of moisturizing lotions. C) Avoid hot-water bottles and heating pads. D) Dry feet vigorously after each bath.

C) Avoid hot-water bottles and heating pads.

The nurse is assessing a patient diagnosed with Grave's disease. What physical characteristics of Grave's disease would the nurse expect to find? A) Hair loss B) Moon face C) Bulging eyes D) Fatigue

C) Bulging eyes

A patient has been assessed for aldosteronism and has recently begun treatment. What are priority areas for assessment that the nurse should frequently address? Select all that apply. A) Pupillary response B) Creatine and BUN levels C) Potassium level D) Peripheral pulses E) BP

C) Creatine and BUN levels E) BP

A patient newly diagnosed with type 2 diabetes is attending a nutrition class. What general guideline would be important to teach the patients at this class? A) Low fat generally indicates low sugar. B) Protein should constitute 30% to 40% of caloric intake. C) Most calories should be derived from carbohydrates. D) Animal fats should be eliminated from the diet.

C) Most calories should be derived from carbohydrates.

The nurse is caring for a patient with a diagnosis of Addison's disease. What sign or symptom is most closely associated with this health problem? A) Truncal obesity B) Hypertension C) Muscle weakness D) Moon face

C) Muscle weakness

A 30-year-old female patient has been diagnosed with Cushing's syndrome. What psychosocial nursing diagnosis should the nurse most likely prioritize when planning the patient's care? A) Decisional conflict r/t treatment options B) Spiritual distress r/t changes in cognitive function C) Disturbed body image r/t changes in physical appearance D) Powerlessness r/t disease progression

C) Disturbed body image r/t changes in cognitive function

A nurse is providing health education to an adolescent newly diagnosed with type 1 diabetes mellitus and her family. The nurse teaches the patient and family that which of the following nonpharmacologic measures will decrease the bodys need for insulin? A) Adequate sleep B) Low stimulation C) Exercise D) Low-fat diet

C) Exercise

A student with diabetes tells the school nurse that he is feeling nervous and hungry. The nurse assesses the child and finds he has tachycardia and is diaphoretic with a blood glucose level of 50 mg/dL (2.8 mmol/L). What should the school nurse administer? A) A combination of protein and carbohydrates, such as a small cup of yogurt B) Two teaspoons of sugar dissolved in a cup of apple juice C) Half of a cup of juice, followed by cheese and crackers D) Half a sandwich with a protein-based filling

C) Half of a cup of juice, followed by cheese and crackers

A nurse caring for a patient with diabetes insipidus is reviewing laboratory results. What is an expected uninalysis finding? A) Glucose in the urine B) Albumin in the urine C) Highly dilute urine D) Leukocytes in the urine

C) Highly dilute urine

A patient has returned to the floor after having a thyroidectomy for thyroid cancer. The nurse knows that sometimes during the surgery the parathyroid glands can be injured or removed. What laboratory finding may be an early indication of parathyroid gland injury or removal? A) Hyponatremia B) Hypophosphatemia C) Hypocalcemia D) Hypokalemia

C) Hypocalcemia

A diabetes educator is teaching a patient about type 2 diabetes. The educator recognizes that the patient understands the primary treatment for type 2 diabetes when the patient states what? A) I read that a pancreas transplant will provide a cure for my diabetes. B) I will take my oral antidiabetic agents when my morning blood sugar is high. C) I will make sure to follow the weight loss plan designed by the dietitian. D) I will make sure I call the diabetes educator when I have questions about my insulin.

C) I will make sure to follow the weight loss plan designed by the dietitian.

A patient has been admitted to the post-surgical unit following a thyroidectomy. To promote comfort and safety, how should the nurse best position the patient? A) Side-lying (lateral) with one pillow under the head B) Head of the bed elevated 30 degrees and no pillows placed under the head C) Semi-Fowler's with the head supported on two pillows D) Supine, with a small roll supporting the neck

C) Semi-Fowler's with the head supported on two pillows

The nurse is teaching a patient that the body needs iodine for the thyroid to function. What food would be the best source of iodine for the body? A) Eggs B) Shellfish C) Table salt D) Red meat

C) Table salt

A physician has explained to a patient that he has developed diabetic neuropathy in his right foot. Later that day, the patient asks the nurse what causes diabetic neuropathy. What would be the nurses best response? A) Research has shown that diabetic neuropathy is caused by fluctuations in blood sugar that have gone on for years. B) The cause is not known for sure but it is thought to have something to do with ketoacidosis. C) The cause is not known for sure but it is thought to involve elevated blood glucose levels over a period of years. D) Research has shown that diabetic neuropathy is caused by a combination of elevated glucose levels and elevated ketone levels.

C) The cause is not known for sure but it is thought to involve elevated blood glucose levels over a period of years.

A 28-year-old pregnant woman is spilling sugar in her urine. The physician orders a glucose tolerance test, which reveals gestational diabetes. The patient is shocked by the diagnosis, stating that she is conscientious about her health, and asks the nurse what causes gestational diabetes. The nurse should explain that gestational diabetes is a result of what etiologic factor? A) Increased caloric intake during the first trimester B) Changes in osmolality and fluid balance C) The effects of hormonal changes during pregnancy D) Overconsumption of carbohydrates during the first two trimesters

C) The effects of hormonal changes during pregnancy

A patient is prescribed corticosteriod therapy. What would be priority information for the nurse to give the patient who is prescribed long-term corticosteroid therapy? A) The patient's diet should be low protein with ample fat. B) The patient may experience short-term changes in cognition. C) The patient is at an increased risk for developing infection. D) The patient is at a decreased risk for development of thrombophelbitis and thromboembolism.

C) The patient is at an increased risk for developing infection.

A patient with type 1 diabetes has told the nurse that his most recent urine test for ketones was positive. What is the nurses most plausible conclusion based on this assessment finding? A) The patient should withhold his next scheduled dose of insulin. B) The patient should promptly eat some protein and carbohydrates. C) The patients insulin levels are inadequate. D) The patient would benefit from a dose of metformin (Glucophage)

C) The patients insulin levels are inadequate.

38. A patient has been assessed for aldosteronism and has recently begun treatment. What are priority areas for assessment that the nurse should frequently address? Select all that apply. A) Pupillary response B) Creatinine and BUN levels C) Potassium level D) Peripheral pulses E) BP

C, E Feedback: Patients with aldosteronism exhibit a profound decline in the serum levels of potassium, and hypertension is the most prominent and almost universal sign of aldosteronism. Pupillary response, peripheral pulses, and renal function are not directly affected.

A nurse is preparing to perform the physical assessment of a newly admitted patient. During which of the following components of the assessment should the nurse wear gloves? Select all that apply. A) Palpation of the patients scalp B) Palpation of the patients upper extremities C) Palpation of a rash on the patients trunk D) Palpation of a lesion on the patients upper back E) Palpation of the patients fingers

C.D

The nurse is preparing a patient for surgery. The patient states that she is very nervous and really does not understand what the surgical procedure is for or how it will be performed. What is the most appropriate nursing action for the nurse to take? Have the patient sign the informed consent and place it in the chart. Call the physician to review the procedure with the patient. Explain the procedure clearly to the patient and her family. Provide the patient with a pamphlet explaining the procedure.

Call the physician to review the procedure with the patient.

The surgical nurse is preparing to send a patient from the presurgical area to the OR and is reviewing the patients informed consent form. What are the criteria for legally valid informed consent? Select all that apply. Consent must be freely given. Consent must be notarized. Consent must be signed on the day of surgery. Consent must be obtained by a physician. Signature must be witnessed by a professional staff member.

Consent must be freely given. Consent must be obtained by a physician. Signature must be witnessed by a professional staff member.

A nurse is assessing a child who has a diagnosis of muscular dystrophy. Assessment reveals that the childs muscles have greater-than-normal tone. The nurse should document the presence of which of the following? A) Tonus B) Flaccidity C) Atony D) Spasticity

D

A nurse is assessing the skin of a patient who has been diagnosed with bacterial cellulitis on the dorsal portion of the great toe. When reviewing the patients health history, the nurse should identify what comorbidity as increasing the patients vulnerability to skin infections? A) Chronic obstructive pulmonary disease B) Rheumatoid arthritis C) Gout D) Diabetes

D

A nurse is caring for a patient who has an MRI scheduled. What is the priority safety action prior to this diagnostic procedure? A) Assessing the patient for signs and symptoms of active infection B) Ensuring that the patient can remain immobile for up to 3 hours C) Assessing the patient for a history of nut allergies D) Ensuring that there are no metal objects on or in the patient

D

A nurse is performing a musculoskeletal assessment of a patient with arthritis. During passive range-of- motion exercises, the nurse hears an audible grating sound. The nurse should document the presence of which of the following? A) Fasciculations B) Clonus C) Effusion D) Crepitus

D

A nurse is providing an educational presentation addressing the topic of Protecting Your Skin. When discussing the anatomy of the skin with this group, the nurse should know that what cells are responsible for producing the pigmentation of the skin? A) Islets of Langerhans B) Squamous cells C) T cells D) Melanocytes

D

A nurse is working with a patient who has a diagnosis of Cushing syndrome. When completing a physical assessment, the nurse should specifically observe for what integumentary manifestation? A) Alopecia B) Yellowish skin tone C) Patchy, bronze pigmentation D) Hirsutism

D

A patient has had a cast placed for the treatment of a humeral fracture. The nurses most recent assessment shows signs and symptoms of compartment syndrome. What is the nurses most appropriate action? A) Arrange for a STAT assessment of the patients serum calcium levels. B) Perform active range of motion exercises. C) Assess the patients joint function symmetrically. D) Contact the primary care provider immediately.

D

A patient with an exceptionally low body mass index has been admitted to the emergency department with signs and symptoms of hypothermia. The nurse should know that this patients susceptibility to heat loss is related to atrophy of what skin component? A) Epidermis B) Merkel cells C) Dermis D) Subcutaneous tissue

D

A patients health assessment has resulted in a diagnosis of alopecia areata. What nursing diagnosis should the nurse most likely associate with this health problem? A) Chronic Pain B) Impaired Skin Integrity C) Impaired Tissue Integrity D) Disturbed Body Image

D

A young student comes to the school nurse and shows the nurse a mosquito bite. As the nurse expects, the bite is elevated and has serous fluid contained in the dermis. How would the nurse classify this lesion? A) Vesicle B) Macule C) Nodule D) Wheal

D

An 82-year-old patient is being treated in the hospital for a sacral pressure ulcer. What age-related change is most likely to affect the patients course of treatment? A) Increased thickness of the subcutaneous skin layer B) Increased vascular supply to superficial skin layers C) Changes in the character and quantity of bacterial skin flora D) Increased time required for wound healing

D

An African American is admitted to the medical unit with liver disease. To correctly assess this patient for jaundice, on what body area should the nurse look for yellow discoloration? A) Elbows B) Lips C) Nail beds D) Sclerae

D

An older adult patient has come to the clinic for a regular check-up. The nurses initial inspection reveals an increased thoracic curvature of the patients spine. The nurse should document the presence of which of the following? A) Scoliosis B) Epiphyses C) Lordosis D) Kyphosis

D

The nurses musculoskeletal assessment of a patient reveals involuntary twitching of muscle groups. How would the nurse document this observation in the patients chart? A) Tetany B) Atony C) Clonus D) Fasciculations

D

While assessing a dark-skinned patient at the clinic, the nurse notes the presence of patchy, milky white spots. The nurse knows that this finding is characteristic of what diagnosis? A) Cyanosis B) Addisons disease C) Polycythemia D) Vitiligo

D

While assessing a patient, the patient tells the nurse that she is experiencing rhythmic muscle contractions when the nurse performs passive extension of her wrist. What is this pattern of muscle contraction referred to as? A) Fasciculations B) Contractures C) Effusion D) Clonus

D

27. A patient has been taking prednisone for several weeks after experiencing a hypersensitivity reaction. To prevent adrenal insufficiency, the nurse should ensure that the patient knows to do which of the following? A) Take the drug concurrent with levothyroxine (Synthroid). B) Take each dose of prednisone with a dose of calcium chloride. C) Gradually replace the prednisone with an OTC alternative. D) Slowly taper down the dose of prednisone, as ordered.

D Feedback: Corticosteroid dosages are reduced gradually (tapered) to allow normal adrenal function to return and to prevent steroid-induced adrenal insufficiency. There are no OTC substitutes for prednisone and neither calcium chloride nor levothyroxine addresses the risk of adrenal insufficiency.

19. A patient with hypofunction of the adrenal cortex has been admitted to the medical unit. What would the nurse most likely find when assessing this patient? A) Increased body temperature B) Jaundice C) Copious urine output D) Decreased BP

D Feedback: Decreased BP may occur with hypofunction of the adrenal cortex. Decreased function of the adrenal cortex does not affect the patients body temperature, urine output, or skin tone.

30. A patient with pheochromocytoma has been admitted for an adrenalectomy to be performed the following day. To prevent complications, the nurse should anticipate preoperative administration of which of the following? A) IV antibiotics B) Oral antihypertensives C) Parenteral nutrition D) IV corticosteroids

D Feedback: IV administration of corticosteroids (methylprednisolone sodium succinate [Solu-Medrol]) may begin on the evening before surgery and continue during the early postoperative period to prevent adrenal insufficiency. Antibiotics, antihypertensives, and parenteral nutrition do not prevent adrenal insufficiency or other common complications of adrenalectomy.

24. A patient with a recent diagnosis of hypothyroidism is being treated for an unrelated injury. When administering medications to the patient, the nurse should know that the patients diminished thyroid function may have what effect? A) Anaphylaxis B) Nausea and vomiting C) Increased risk of drug interactions D) Prolonged duration of effect

D Feedback: In all patients with hypothyroidism, the effects of analgesic agents, sedatives, and anesthetic agents are prolonged. There is no direct increase in the risk of anaphylaxis, nausea, or drug interactions, although these may potentially result from the prolonged half-life of drugs.

12. The home care nurse is conducting patient teaching with a patient on corticosteroid therapy. To achieve consistency with the bodys natural secretion of cortisol, when would the home care nurse instruct the patient to take his or her corticosteroids? A) In the evening between 4 PM and 6 PM B) Prior to going to sleep at night C) At noon every day D) In the morning between 7 AM and 8 AM

D Feedback: In keeping with the natural secretion of cortisol, the best time of day for the total corticosteroid dose is in the morning from 7 to 8 AM. Large-dose therapy at 8 AM, when the adrenal gland is most active, produces maximal suppression of the gland. Also, a large 8 AM dose is more physiologic because it allows the body to escape effects of the steroids from 4 PM to 6 AM, when serum levels are normally low, thus minimizing cushingoid effects.

14. The nurse is caring for a patient with hyperparathyroidism. What level of activity would the nurse expect to promote? A) Complete bed rest B) Bed rest with bathroom privileges C) Out of bed (OOB) to the chair twice a day D) Ambulation and activity as tolerated

D Feedback: Mobility, with walking or use of a rocking chair for those with limited mobility, is encouraged as much as possible because bones subjected to normal stress give up less calcium. Best rest should be discouraged because it increases calcium excretion and the risk of renal calculi. Limiting the patient to getting out of bed only a few times a day also increases calcium excretion and the associated risks.

40. The nurse is providing care for an older adult patient whose current medication regimen includes levothyroxine (Synthroid). As a result, the nurse should be aware of the heightened risk of adverse effects when administering an IV dose of what medication? A) A fluoroquinalone antibiotic B) A loop diuretic C) A proton pump inhibitor (PPI) D) A benzodiazepine

D Feedback: Oral thyroid hormones interact with many other medications.Even in small IV doses, hypnotic and sedative agents may induce profound somnolence, lasting far longer than anticipated and leading to narcosis (stupor like condition). Furthermore, they are likely to cause respiratory depression, which can easily be fatal because of decreased respiratory reserve and alveolar hypoventilation. Antibiotics, PPIs and diuretics do not cause the same risk.

34. A patient on corticosteroid therapy needs to be taught that a course of corticosteroids of 2 weeks duration can suppress the adrenal cortex for how long? A) Up to 4 weeks B) Up to 3 months C) Up to 9 months D) Up to 1 year

D Feedback: Suppression of the adrenal cortex may persist up to 1 year after a course of corticosteroids of only 2 weeks duration.

A diabetes nurse educator is presenting the American Diabetes Association (ADA) recommendations for levels of caloric intake. What do the ADAs recommendations include? A) 10% of calories from carbohydrates, 50% from fat, and the remaining 40% from protein B) 10% to 20% of calories from carbohydrates, 20% to 30% from fat, and the remaining 50% to 60% from protein C) 20% to 30% of calories from carbohydrates, 50% to 60% from fat, and the remaining 10% to 20% from protein D) 50% to 60% of calories from carbohydrates, 20% to 30% from fat, and the remaining 10% to 20% from protein

D) 50% to 60% of calories from carbohydrates, 20% to 30% from fat, and the remaining 10% to 20% from protein

The nurse is providing care for an older adult patient whose current medication regimen includes levothyroxine (Synthroid). As a result, the nurse should be aware of the heightened risk of adverse effects when administering an IV dose of what medication? A) A fluroquinalone antibiotic B) A loop diuretic C) A proton pump inhibitor (PPI) D) A benzodiazepine

D) A benzodiazepine

The nurse is caring for a patient with hyperparathyroidism. What level of activity would the nurse expect to promote? A) Complete bed rest B) Bed rest with bathroom privileges C) Out of bed (OOB) to the chair twice a day D) Ambulation and activity as tolerated

D) Ambulation and activity as tolerated

A patient has received a diagnosis of type 2 diabetes. The diabetes nurse has made contact with the patient and will implement a program of health education. What is the nurses priority action? A) Ensure that the patient understands the basic pathophysiology of diabetes. B) Identify the patients body mass index. C) Teach the patient survival skills for diabetes. D) Assess the patients readiness to learn.

D) Assess the patients readiness to learn.

A patient with hypofunction of the adrenal cortex has been admitted to the medical unit. What would the nurse most likely find when assessing this patient? A) Increased body temperature B) Jaundice C) Copious urine output D) Decreased BP

D) Decreased BP

A newly admitted patient with type 1 diabetes asks the nurse what caused her diabetes. When the nurse is explaining to the patient the etiology of type 1 diabetes, what process should the nurse describe? A) The tissues in your body are resistant to the action of insulin, making the glucose levels in your blood increase. B) Damage to your pancreas causes an increase in the amount of glucose that it releases, and there is not enough insulin to control it. C) The amount of glucose that your body makes overwhelms your pancreas and decreases your production of insulin. D) Destruction of special cells in the pancreas causes a decrease in insulin production. Glucose levels rise because insulin normally breaks it down.

D) Destruction of special cells in the pancreas causes a decrease in insulin production. Glucose levels rise because insulin normally breaks it down.

An older adult patient with type 2 diabetes is brought to the emergency department by his daughter. The patient is found to have a blood glucose level of 623 mg/dL. The patients daughter reports that the patient recently had a gastrointestinal virus and has been confused for the last 3 hours. The diagnosis of hyperglycemic hyperosmolar syndrome (HHS) is made. What nursing action would be a priority? A) Administration of antihypertensive medications B) Administering sodium bicarbonate intravenously C) Reversing acidosis by administering insulin D) Fluid and electrolyte replacement

D) Fluid and electrolyte replacement

A patient with pheochromocytoma has been admitted for an adrenalectomy to be performed the following day. To prevent complications, the nurse should anticipate preoperative administration of which of the following? A) IV antibiotics B) Oral antihypertensives C) Parenteral nutrition D) IV corticosteroids

D) IV corticosteroids

The home care nurse is conducting patient teaching with a patient on corticosteroid therapy. To achieve consistency with the body's natural secretion of cortisol, when would the home care nurse instruct the patient to take his or her corticosteroids? A) Un the evening between 4PM and 6PM B) Prior to going to sleep at night C) At noon every day D) In the morning between 7AM and 8AM

D) In the morning between 7AM and 8AM

A school nurse is teaching a group of high school students about risk factors for diabetes. Which of the following actions has the greatest potential to reduce an individuals risk for developing diabetes? A) Have blood glucose levels checked annually. B) Stop using tobacco in any form. C) Undergo eye examinations regularly. D) Lose weight, if obese.

D) Lose weight, if obese.

A medical nurse is aware of the need to screen specific patients for their risk of hyperglycemic hyperosmolar syndrome (HHS). In what patient population does hyperosmolar nonketotic syndrome most often occur? A) Patients who are obese and who have no known history of diabetes B) Patients with type 1 diabetes and poor dietary control C) Adolescents with type 2 diabetes and sporadic use of antihyperglycemics D) Middle-aged or older people with either type 2 diabetes or no known history of diabetes

D) Middle-aged or older people with either type 2 diabetes or no known history of diabetes

A patient with a recent diagnosis of hypothyroidism is being treated for an unrelated injury. When administering medications to the patient, the nurse should know that the patient's diminished thyroid function may have what effect? A) Anaphylaxis B) Nausea and vomiting C) Increased risk of drug interactions D) Prolonged duration of effect

D) Prolonged duration of effect

A nurse is teaching basic survival skills to a patient newly diagnosed with type 1 diabetes. What topic should the nurse address? A) Signs and symptoms of diabetic nephropathy B) Management of diabetic ketoacidosis C) Effects of surgery and pregnancy on blood sugar levels D) Recognition of hypoglycemia and hyperglycemia

D) Recognition of hypoglycemia and hyperglycemia

A patient with type 2 diabetes has been managing his blood glucose levels using diet and metformin (Glucophage). Following an ordered increase in the patients daily dose of metformin, the nurse should prioritize which of the following assessments? A) Monitoring the patients neutrophil levels B) Assessing the patient for signs of impaired liver function C) Monitoring the patients level of consciousness and behavior D) Reviewing the patients creatinine and BUN levels

D) Reviewing the patients creatinine and BUN levels

A patient has been taking prednisone for several weeks after experiencing a hypersensitivity reaction. To prevent adrenal insufficiency, the nurse should ensure that the patient knows to do which of the following? A) Take the drug concurrent with levothyroxine (Synthroid) B) Take each dose of prednisone with a dose of calcium C) Gradually replace the prednisone with an OTC alternative D) Slowly taper down the dose of prednisone, as ordered

D) Slowly taper down the dose of prednisone, as ordered

A nurse is assessing a patient who has diabetes for the presence of peripheral neuropathy. The nurse should question the patient about what sign or symptom that would suggest the possible development of peripheral neuropathy? A) Persistently cold feet B) Pain that does not respond to analgesia C) Acute pain, unrelieved by rest D) The presence of a tingling sensation

D) The presence of a tingling sensation

A patient on corticosteroid therapy needs to be taught that a course of corticosteroids of 2 weeks' duration can suppress the adrenal cortex for how long? A) Up to 4 weeks B) Up to 3 months C) Up to 9 months D) Up to 1 year

D) Up to 1 year

The nurse is caring for a patient who is admitted to the ER with the diagnosis of acute appendicitis. The nurse notes during the assessment that the patients ribs and xiphoid process are prominent. The patient states she exercises two to three times daily and her mother indicates that she is being treated for anorexia nervosa. How should the nurse best follow up these assessment data? Inform the postoperative team about the patients risk for wound dehiscence. Evaluate the patients ability to manage her pain level. Facilitate a detailed analysis of the patients electrolyte levels. Instruct the patient on the need for a high-sodium diet to promote healing.

Facilitate a detailed analysis of the patients electrolyte levels.

A nurse is aware that the outer layer of the skin consists of dead cells that contain large amounts of keratin. The physiologic functions of keratin include which of the following? Select all that apply. A) Producing antibodies B) Absorbing electrolytes C) Maintaining acidbase balance D) Physically repelling pathogens E) Preventing fluid loss

D,E

The nurse is doing a preoperative assessment of an 87-year-old man who is slated to have a right lung lobe resection to treat lung cancer. What underlying principle should guide the nurses preoperative assessment of an elderly patient? Elderly patients have a smaller lung capacity than younger patients. Elderly patients require higher medication doses than younger patients. Elderly patients have less physiologic reserve than younger patients. Elderly patients have more sophisticated coping skills than younger patients.

Elderly patients have less physiologic reserve than younger patients.

A nurse in the preoperative holding area is admitting a woman prior to reduction mammoplasty. What should the nurse include in the care given to this patient? Select all that apply. Establishing an IV line Verifying the surgical site with the patient Taking measures to ensure the patients comfort Applying a grounding device to the patient Preparing the medications to be administered in the OR

Establishing an IV line Verifying the surgical site with the patient Taking measures to ensure the patients comfort

A patient comes to the clinic complaining of a red rash of small, fluid-filled blisters and is suspected of having herpes zoster. What presentation is most consistent with herpes zoster? A) Grouped vesicles occurring on lips and oral mucous membranes B) Grouped vesicles occurring on the genitalia C) Rough, fresh, or gray skin protrusions D) Grouped vesicles in linear patches along a dermatome

Grouped vesicles in linear patches along a dermatome

The nurse is planning the care of a patient who has type 1 diabetes and who will be undergoing knee replacement surgery. This patients care plan should reflect an increased risk of what postsurgical complications? Select all that apply. Hypoglycemia Delirium Acidosis Glucosuria Fluid overload

Hypoglycemia Acidosis Glucosuria

When writing a plan of care for a patient with psoriasis, the nurse would know that an appropriate nursing diagnosis for this patient would be what A) Impaired Skin Integrity Related to Scaly Lesions B) Acute Pain Related to Blistering and Erosions of the Oral Cavity C) Impaired Tissue Integrity Related to Epidermal Shedding D) Anxiety Related to Risk for Melanoma

Impaired Skin Integrity Related to Scaly Lesions An appropriate diagnosis for a patient with psoriasis would include Impaired Skin Integrity as it relates to scaly lesions.

The nurse is creating the care plan for a 70-year-old obese patient who has been admitted to the postsurgical unit following a colon resection. This patients age and increased body mass index mean that she is at increased risk for what complication in the postoperative period? Hyperglycemia Azotemia Falls Infection

Infection

You are providing preoperative teaching to a patient scheduled for hip replacement surgery in 1 month. During the preoperative teaching, the patient gives you a list of medications she takes, the dosage, and frequency. Which of the following interventions provides the patient with the most accurate information? Instruct the patient to stop taking St. Johns wort at least 2 weeks prior to surgery due to its interaction with anesthetic agents. Instruct the patient to continue taking ephedrine prior to surgery due to its beneficial effect on blood pressure. Instruct the patient to discontinue Synthroid due to its effect on blood coagulation and the potential for heart dysrhythmias. Instruct the patient to continue any herbal supplements unless otherwise instructed, and inform the patient that these supplements have minimal effect on the surgical procedure.

Instruct the patient to stop taking St. Johns wort at least 2 weeks prior to surgery due to its interaction with anesthetic agents.

The nurse is preparing to send a patient to the OR for a scheduled surgery. What should the nurse ensure is on the chart when it accompanies the patient to surgery? Select all that apply. Laboratory reports Nurses notes Verification form Social work assessment Dieticians assessment

Laboratory reports Nurses notes Verification form

A nurse is providing preoperative teaching to a patient who will soon undergo a cardiac bypass. The nurses teaching plan includes exercises of the extremities. What is the purpose of teaching a patient leg exercises prior to surgery? Leg exercises increase the patients muscle mass postoperatively. Leg exercises improve circulation and prevent venous thrombosis. Leg exercises help to prevent pressure sores to the sacrum and heels. Leg exercise help increase the patients level of consciousness after surgery.

Leg exercises improve circulation and prevent venous thrombosis.

While performing an initial assessment of a patient admitted with appendicitis, the nurse observes an elevated blue-black lesion on the patient's ear. The nurse knows that this lesion is consistent with what type of skin cancer? A) Basal cell carcinoma B) Squamous cell carcinoma C) Dermatofibroma D) Malignant melanoma

Malignant melanoma presents itself as a superficial spreading melanoma which may appear in a combination of colors, with hues of tan, brown, and black mixed with gray, blue-black, or white.

An OR nurse will be participating in the intraoperative phase of a patients kidney transplant. What action will the nurse prioritize in this aspect of nursing care? Monitoring the patients physiologic status Providing emotional support to family Maintaining the patients cognitive status Maintaining a clean environment

Monitoring the patients physiologic status

During the care of a preoperative patient, the nurse has given the patient a preoperative benzodiazepine. The patient is now requesting to void. What action should the nurse take? Assist the patient to the bathroom. Offer the patient a bedpan or urinal. Wait until the patient gets to the operating room and is catheterized. Have the patient go to the bathroom.

Offer the patient a bedpan or urinal.

The nurse is caring for a hospice patient who is scheduled for a surgical procedure to reduce the size of his spinal tumor in an effort to relieve his pain. The nurse should plan this patient care with the knowledge that his surgical procedure is classified as which of the following? Diagnostic Laparoscopic Curative Palliative

Palliative

A patient diagnosed with a stasis ulcer has been hospitalized. There is an order to change the dressing and provide wound care. Which activity should the nurse first perform when providing wound care? A) Assess the drainage in the dressing. B) Slowly remove the soiled dressing. C) Perform hand hygiene. D) Don non-latex gloves.

Perform hand hygiene.

A patient is on call to the OR for an aortobifemoral bypass and the nurse administers the ordered preoperative medication. After administering a preoperative medication to the patient, what should the nurse do? Encourage light ambulation. Place the bed in a low position with the side rails up. Tell the patient that he will be asleep before he leaves for surgery. Take the patients vital signs every 15 minutes.

Place the bed in a low position with the side rails up.

A patient is scheduled for a bowel resection in the morning and the patients orders include a cleansing enema tonight. The patient wants to know why this is necessary. The nurse should explain that the cleansing enema will have what therapeutic effect? Preventing aspiration of gastric contents Preventing the accumulation of abdominal gas postoperatively Preventing potential contamination of the peritoneum Facilitating better absorption of medications

Preventing potential contamination of the peritoneum

A 90-year-old female patient is scheduled to undergo a partial mastectomy for the treatment of breast cancer. What nursing diagnosis should the nurse prioritize when planning this patients postoperative care? Risk for Delayed Growth and Development related to prolonged hospitalization Risk for Decisional Conflict related to discharge planning Risk for Impaired Memory related to old age Risk for Infection related to reduced immune function

Risk for Infection related to reduced immune function

The nurse is doing preoperative patient education with a 61-year-old male patient who has a 40-pack per year history of cigarette smoking. The patient will undergo an elective bunionectomy at a time that fits his work schedule in a few months. What would be the best instruction to give to this patient? Reduce smoking by 50% to prevent the development of pneumonia. Stop smoking at least 6 weeks before the scheduled surgery to enhance pulmonary function and decrease infection. Aim to quit smoking in the postoperative period to reduce the chance of surgical complications Stop smoking 4 to 8 weeks before the scheduled surgery to enhance pulmonary function and decrease infection.

Stop smoking 4 to 8 weeks before the scheduled surgery to enhance pulmonary function and decrease infection.

The ED nurse is caring for an 11-year-old brought in by ambulance after having been hit by a car. The childs parents are thought to be en route to the hospital but have not yet arrived. No other family members are present and attempts to contact the parents have been unsuccessful. The child needs emergency surgery to save her life. How should the need for informed consent be addressed? A social worker should temporarily sign the informed consent. Consent should be obtained from the hospitals ethics committee. Surgery should be done without informed consent. Surgery should be delayed until the parents arrive.

Surgery should be done without informed consent.

A patient with squamous cell carcinoma has been scheduled for treatment of this malignancy. The nurse should anticipate that treatment for this type of cancer will primarily consist of what intervention? A) Chemotherapy B) Radiation therapy C) Surgical excision D) Biopsy of sample tissue

Surgical excision The primary goal of surgical management of squamous cell carcinoma is to remove the tumor entirely.

The admitting nurse in a short-stay surgical unit is responsible for numerous aspects of care. What must the nurse verify before the patient is taken to the preoperative holding area? That preoperative teaching was performed That the family is aware of the length of the surgery That follow-up home care is not necessary That the family understands the patient will be discharged immediately after surgery.

That preoperative teaching was performed

The nurse admitting a patient who is insulin dependent to the same-day surgical suite for carpal tunnel surgery. How should this patients diagnosis of type 1 diabetes affect the care that the nurse plans? The nurse should administer a bolus of dextrose IV solution preoperatively. The nurse should keep the patient NPO for at least 8 hours preoperatively. The nurse should initiate a subcutaneous infusion of long-acting insulin. The nurse should assess the patients blood glucose levels vigilantly.

The nurse should assess the patients blood glucose levels vigilantly.

in anticipation of a patients scheduled surgery, the nurse is teaching her to perform deep breathing and coughing to use postoperatively. What action should the nurse teach the patient? The patient should take three deep breaths and cough hard three times, at least every 15 minutes for the immediately postoperative period. The patient should take three deep breaths and exhale forcefully and then take a quick short breath and cough from deep in the lungs. The patient should take a deep breath in through the mouth and exhale through the mouth, take a short breath, and cough from deep in the lungs. The patient should rapidly inhale, hold for 30 seconds or as long as possible, and exhale slowly.

The patient should take a deep breath in through the mouth and exhale through the mouth, take a short breath, and cough from deep in the lungs.

The PACU nurse is caring for a patient who has been deemed ready to go to the postsurgical floor after her surgery. What would the PACU nurse be responsible for reporting to the nurse on the floor? Select all that apply. The names of the anesthetics that were used The identities of the staff in the OR The patients preoperative level of consciousness The presence of family and/or significant others The patients full name

The patients preoperative level of consciousness The presence of family and/or significant others The patients full name

The nurse is preparing a patient for surgery prior to her hysterectomy without oophorectomy. The nurse is witnessing the patients signature on a consent form. Which comment by the patient would best indicate informed consent? I know Ill be fine because the physician said he has done this procedure hundreds of times. I know Ill have pain after the surgery but theyll do their best to keep it to a minimum. The physician is going to remove my uterus and told me about the risk of bleeding. Because the physician isnt taking my ovaries, Ill still be able to have children.

The physician is going to remove my uterus and told me about the risk of bleeding.

A patient requires a full-thickness graft to cover a chronic wound. How is the donor site selected? A) The largest area of the body without hair is selected. B) Any area that is not normally visible can be used. C) An area matching the color and texture of the skin at the surgical site is selected. D) An area matching the sensory capability of the skin at the surgical site is selected.

The site where the intact skin is harvested is called the donor site. Selection of the donor site is made to match the color and texture of skin at the surgical site and to leave as little scarring as possible.

You are the nurse caring for an unconscious trauma victim who needs emergency surgery. The patient is a 55-year-old man with an adult son. He is legally divorced and is planning to be remarried in a few weeks. His parents are at the hospital with the other family members. The physician has explained the need for surgery, the procedure to be done, and the risks to the children, the parents, and the fianc. Who should be asked to sign the surgery consent form? The fianc The son The physician, acting as a surrogate The patients father

The son

You are caring for an 88-year-old woman who is scheduled for a right mastectomy. You know that elderly patients are frequently more anxious prior to surgery than younger patients. What would you increase with this patient to decrease her anxiety? Analgesia Therapeutic touch Preoperative medication Sleeping medication the night before surgery

Therapeutic touch

The nurse is providing preoperative teaching to a patient scheduled for surgery. The nurse is instructing the patient on the use of deep breathing, coughing, and the use of incentive spirometry when the patient states, I dont know why youre focusing on my breathing. My surgery is on my hip, not my chest. What rationale for these instructions should the nurse provide? To prevent chronic obstructive pulmonary disease (COPD) To promote optimal lung expansion To enhance peripheral circulation To prevent pneumothorax

To promote optimal lung expansion

A nurse is caring for a patient who has been diagnosed with psoriasis. The nurse is creating an education plan for the patient. What information should be included in this plan A) Use caution when taking nonprescription medications. B) Avoid public places until symptoms subside. C) Wash skin frequently to prevent infection. D) Liberally apply corticosteroids as needed.

Use caution when taking nonprescription medications. The patient should be cautioned about taking nonprescription medications because some may aggravate mild psoriasis.

The nurse is caring for a patient who is experiencing pain and anxiety following his prostatectomy.Which intervention will likely best assist in decreasing the patients pain and anxiety? Administration of NSAIDs rather than opioids Allowing the patient to increase activity Use of guided imagery along with pain medication Use of deep breathing and coughing exercises

Use of guided imagery along with pain medication

The nurse is performing a preadmission assessment of a patient scheduled for a bilateral mastectomy. Of what purpose of the preadmission assessment should the nurse be aware? Verifies completion of preoperative diagnostic testing Discusses and reviews patients health insurance coverage Determines the patients suitability as a surgical candidate Informs the patient of need for postoperative transportation

Verifies completion of preoperative diagnostic testing

A clinic nurse is conducting a preoperative interview with an adult patient who will soon be scheduled to undergo cardiac surgery. What interview question most directly addresses the patients safety? What prescription and nonprescription medications do you currently take? Have you previously been admitted to the hospital, either for surgery or for medical treatment? How long do you expect to be at home recovering after your surgery? Would you say that you tend to eat a fairly healthy diet?

What prescription and nonprescription medications do you currently take?

A 77-year-old mans coronary artery bypass graft has been successful and discharge planning is underway. When planning the patients subsequent care, the nurse should know that the postoperative phase of perioperative nursing ends at what time? When the patient is returned to his room after surgery When a follow-up evaluation in the clinical or home setting is done When the patient is fully recovered from all effects of the surgery When the family becomes partly responsible for the patients care

When a follow-up evaluation in the clinical or home setting is done

The nurse is caring for a trauma victim in the ED who will require emergency surgery due to injuries. Before the patient leaves the ED for the OR, the patient goes into cardiac arrest. The nurse assists in the successful resuscitation and proceeds to release the patient to the OR staff. When can the ED nurse perform the preoperative assessment? When he or she has the opportunity to review the patients electronic health record When the patient arrives in the OR When assisting with the resuscitation Preoperative assessment is not necessary in this case

When assisting with the resuscitation

A patient is admitted to the ED complaining of severe abdominal pain, stating that he has been vomiting coffee-ground like emesis. The patient is diagnosed with a perforated gastric ulcer and is informed that he needs surgery. When can the patient most likely anticipate that the surgery will be scheduled? Within 24 hours Within the next week Without delay because the bleed is emergent As soon as all the days elective surgeries have been completed

Without delay because the bleed is emergent

The nurse is caring for a patient in the postoperative period following an abdominal hysterectomy. The patient states, I dont want to use my pain meds because theyll make me dependent and I wont get better as fast. Which response is most important when explaining the use of pain medication? You will need the pain medication for at least 1 week to help in your recovery. What do you mean you feel you wont get better faster? Pain medication will help to decrease your pain and increase your ability to breath. Dependency is a risk with pain medication, but you are young and wont have any problems. Pain medication can be given by mouth to prevent the risk of dependency that you are worried about. The pain medication has not been shown to affect your risk of a slowed recovery. You will move more easily and heal more quickly with decreased pain. Dependence only occurs when it is administered for an extended period of time.

You will move more easily and heal more quickly with decreased pain. Dependence only occurs when it is administered for an extended period of time.

One of the things a nurse has taught to a patient during preoperative teaching is to have nothing by mouth for the specified time before surgery. The patient asks the nurse why this is important. What is the most appropriate response for the patient? You will need to have food and fluid restricted before surgery so you are not at risk for choking. The restriction of food or fluid will prevent the development of pneumonia related to decreased lung capacity. The presence of food in the stomach interferes with the absorption of anesthetic agents. By withholding food for 8 hours before surgery, you will not develop constipation in the postoperative period.

You will need to have food and fluid restricted before surgery so you are not at risk for choking.

A 35-year-old kidney transplant patient comes to the clinic exhibiting new skin lesions. The diagnosis is Kaposi's sarcoma. The nurse caring for this patient recognizes that this is what type of Kaposi's sarcoma? A) Classic B) AIDS-related C) Immunosuppression-related D) Endemic

ans: C Feedback: Immunosuppression-associated Kaposi's sarcoma occurs in transplant recipients and people with AIDS. This form of KS is characterized by local skin lesions and disseminated visceral and mucocutaneous diseases. Classic Kaposi's sarcoma occurs predominantly in men of Mediterranean or Jewish ancestry between 40 and 70 years of age. Endemic KS affects people predominantly in the eastern half of Africa. AIDS-related KS is seen in people with AIDS.

The clinic nurse is doing a preoperative assessment of a patient who will be undergoing outpatient cataract surgery with lens implantation in 1 week. While taking the patients medical history, the nurse notes that this patient had a kidney transplant 8 years ago and that the patient is taking immunosuppressive drugs. For what is this patient at increased risk when having surgery? Rejection of the kidney Rejection of the implanted lens Infection Adrenal storm

infection

The nurse is checking the informed consent for a 17-year-old who has just been married and expecting her first child. She is scheduled for a cesarean section. She is still living with her parents and is on her parents health insurance. When obtaining informed consent for the cesarean section, who is legally responsible for signing? Her parents Her husband The patient The obstetrician

the patient


Ensembles d'études connexes

Chapter 5: The Knee and Patellofemoral Joints

View Set

Introduction to Sociology- Soci 1301

View Set

Enzymology Part 2 - Enzyme Kinetics lec

View Set